Rd Sharma 2022 Solutions for Class 10 Maths Chapter 6 Co Ordinate Geometry are provided here with simple step-by-step explanations. These solutions for Co Ordinate Geometry are extremely popular among Class 10 students for Maths Co Ordinate Geometry Solutions come handy for quickly completing your homework and preparing for exams. All questions and answers from the Rd Sharma 2022 Book of Class 10 Maths Chapter 6 are provided here for you for free. You will also love the ad-free experience on Meritnation’s Rd Sharma 2022 Solutions. All Rd Sharma 2022 Solutions for class Class 10 Maths are prepared by experts and are 100% accurate.

Page No 226:

Question 1:

On which axis do the following points lie?

(a) P(5, 0)
 
(b) Q(0−2)

(c) R(−4,0)

(d) S(0,5)

Answer:

According to the Rectangular Cartesian Co-ordinate system of representing a point (x, y),

If then the point lies in the 1st quadrant

If then the point lies in the 2nd quadrant

If then the point lies in the 3rd quadrant

If then the point lies in the 4th quadrant

But in case

If then the point lies on the y-axis

If then the point lies on the x-axis

(i) Here the point is given to be P (5, 0). Comparing this with the standard form of

(x, y) we have

Here we see that

Hence the given point lies on the

(ii) Here the point is given to be Q (0, -2). Comparing this with the standard form of (x, y) we have

Here we see that

Hence the given point lies on the

(iii) Here the point is given to be R (-4, 0). Comparing this with the standard form of (x, y) we have

Here we see that

Hence the given point lies on the

(iv) Here the point is given to be S (0, 5). Comparing this with the standard form of (x, y) we have

Here we see that

Hence the given point lies on the

Page No 226:

Question 2:

Let ABCD be a square of side 2a. Find the coordinates of the vertices of this square when

(i) A coincides with the origin and AB and AD are along OX and OY respectively.
(ii) The centre of the square is at the origin and coordinate axes are parallel to the sides AB and AD respectively.

Answer:

The distance between any two adjacent vertices of a square will always be equal. This distance is nothing but the side of the square.

Here, the side of the square ‘ABCD’ is given to be ‘2a’.

(i) Since it is given that the vertex ‘A’ coincides with the origin we know that the co-ordinates of this point is (0, 0).

We also understand that the side ‘AB’ is along the x-axis. So, the vertex ‘B’ has got to be at a distance of ‘2a’ from ‘A’.

Hence the vertex ‘B’ has the co-ordinates (2a, 0).

Also it is said that the side ‘AD’ is along the y-axis. So, the vertex ‘D’ it has got to be at a distance of ‘2a’ from ‘A’.

Hence the vertex ‘D’ has the co-ordinates (0, 2a)

Finally we have vertex ‘C’ at a distance of ‘2a’ both from vertex ‘B’ as well as ‘D’.

Hence the vertex of ‘C’ has the co-ordinates (2a, 2a)

So, the co-ordinates of the different vertices of the square are

(ii) Here it is said that the centre of the square is at the origin and that the sides of the square are parallel to the axes.

Moving a distance of half the side of the square in either the ‘upward’ or ‘downward’ direction and also along either the ‘right’ or ‘left’ direction will give us all the four vertices of the square.

Half the side of the given square is ‘a’.

The centre of the square is the origin and its vertices are (0, 0). Moving a distance of ‘a’ to the right as well as up will lead us to the vertex ‘A’ and it will have vertices (a, a).

Moving a distance of ‘a’ to the left as well as up will lead us to the vertex ‘B’ and it will have vertices (-(a, a).

Moving a distance of ‘a’ to the left as well as down will lead us to the vertex ‘C’ and it will have vertices (-(a, -a).

Moving a distance of ‘a’ to the right as well as down will lead us to the vertex ‘D’ and it will have vertices (a,-,−a).

So, the co-ordinates of the different vertices of the square are

Page No 226:

Question 3:

The base PQ of two equilateral triangles PQR and PQR' with side 2a lies along y-axis such that the mid-point of PQ is at the origin. Find the coordinates of the vertices R and R' of the triangles.

Answer:

In an equilateral triangle, the height ‘h’ is given by

Here it is given that ‘PQ’ forms the base of two equilateral triangles whose side measures ‘2a’ units.

The height of these two equilateral triangles has got to be

In an equilateral triangle the height drawn from one vertex meets the midpoint of the side opposite this vertex.

So here we have ‘PQ’ being the base lying along the y-axis with its midpoint at the origin, that is at (0, 0).

So the vertices ‘R’ and ‘R’’ will lie perpendicularly to the y-axis on either sides of the origin at a distance of ‘’ units.

Hence the co-ordinates of ‘R’ and ‘R’’ are



Page No 277:

Question 1:

Find the distance between the following pair of points:

(a) (−6, 7) and (−1, −5)

(b) (a+b, b+c) and (ab, cb)

(c) (asinα, −bcosα) and (−acos α, bsin α)

(d) (a, 0) and (0, b)

Answer:

The distance d between two points and is given by the formula

(i) The two given points are (−6, 7) and (−1, −5)

The distance between these two points is

Hence the distance is.

(ii) The two given points are

The distance between these two points is

Hence the distance is.

(iii) The two given points are and

The distance between these two points is

Hence the distance is.

(iv) The two given points are (a, 0) and (0, b)

The distance between these two points is

Hence the distance is.

Page No 277:

Question 2:

Find the value of a when the distance between the points (3, a) and (4, 1) is 10.

Answer:

The distance d between two points and is given by the formula

The distance between two points (3, a) and (4, 1) is given as. Substituting these values in the formula for distance between two points we have,

Now, squaring the above equation on both sides of the equals sign

Thus we arrive at a quadratic equation. Let us solve this now,

The roots of the above quadratic equation are thus 4 and −2.

Thus the value of ‘a’ could either be.

Page No 277:

Question 3:

The length of a line segment is of 10 units and the coordinates of one end-point are (2, −3). If the abscissa of the other end is 10, find the ordinate of the other end.

Answer:

The distance d between two points and is given by the formula

Here it is given that one end of a line segment has co−ordinates (2,−3). The abscissa of the other end of the line segment is given to be 10. Let the ordinate of this point be ‘y’.

So, the co−ordinates of the other end of the line segment is (10, y).

The distance between these two points is given to be 10 units.

Substituting these values in the formula for distance between two points we have,

Squaring on both sides of the equation we have,

We have a quadratic equation for ‘y’. Solving for the roots of this equation we have,

The roots of the above equation are ‘9’ and ‘3’

Thus the ordinates of the other end of the line segment could be.

Page No 277:

Question 4:

Show that the points (−4, −1), (−2, −4) (4, 0) and (2, 3) are the vertices points of a rectangle.

Answer:

The distance d between two points and is given by the formula

In a rectangle, the opposite sides are equal in length. The diagonals of a rectangle are also equal in length.

Here the four points are A(4,1), B(2,4), C(4,0) and D(2,3).

First let us check the length of the opposite sides of the quadrilateral that is formed by these points.

We have one pair of opposite sides equal.

Now, let us check the other pair of opposite sides.

The other pair of opposite sides are also equal. So, the quadrilateral formed by these four points is definitely a parallelogram.

For a parallelogram to be a rectangle we need to check if the diagonals are also equal in length.

Now since the diagonals are also equal we can say that the parallelogram is definitely a rectangle.

Hence we have proved that the quadrilateral formed by the four given points is a.

Page No 277:

Question 5:

Show that the points A (1, −2), B (3, 6), C (5, 10) and D (3, 2) are the vertices of a parallelogram.

Answer:

The distance d between two points and is given by the formula

In a parallelogram the opposite sides are equal in length.

Here the four points are A(1,2), B(3, 6), C(5, 10) and D(3, 2).

Let us check the length of the opposite sides of the quadrilateral that is formed by these points.

We have one pair of opposite sides equal.

Now, let us check the other pair of opposite sides.

The other pair of opposite sides is also equal. So, the quadrilateral formed by these four points is definitely a parallelogram.

Hence we have proved that the quadrilateral formed by the given four points is a .

 

Page No 277:

Question 6:

Show that ΔABC, where A(–2, 0), B(2, 0), C(0, 2) and ΔPQR where P(–4, 0), Q(4, 0), R(0, 2) are similar triangles.

Answer:

In ΔABC, the coordinates of the vertices are A(–2, 0), B(2, 0), C(0, 2). 

AB=2+22+0-02=4BC=0-22+2-02=8=22CA=0+22+2-02=8=22

In ΔPQR, the coordinates of the vertices are P(–4, 0), Q(4, 0), R(0, 4).

PQ=4+42+0-02=8QR=0-42+4-02=42PR=0+42+4-02=42

Now, for ΔABC and ΔPQR to be similar, the corresponding sides should be proportional. 

So, ABPQ=BCQR=CAPR48=2242=2242=12
Thus, ΔABC is similar to ΔPQR. 

 

Page No 277:

Question 7:

Prove that the points (3, 0), (6, 4) and (−1, 3) are vertices of a right-angled isosceles triangle.

Answer:

The distance d between two points and is given by the formula

In an isosceles triangle there are two sides which are equal in length.

Here the three points are A(3, 0), B(6, 4) and C(1, 3).

Let us check the length of the three sides of the triangle.

Here, we see that two sides of the triangle are equal. So the triangle formed should be an isosceles triangle.

We can also observe that

Hence proved that the triangle formed by the three given points is an.

Page No 277:

Question 8:

Prove that (2, −2) (−2, 1) and (5, 2) are the vertices of a right angled triangle. Find the area of the triangle and the length of the hypotenuse.

Answer:

The distance d between two points and is given by the formula

In a right-angled triangle, by Pythagoras theorem, the square of the longest side is equal to the sum of squares of the other two sides in the triangle.

Here the three points are A(2,2), B(2,1) and C(5,2).

Let us find out the lengths of all the sides of the triangle.

Here we have,

Since the square of the longest side is equal to the sum of squares of the other two sides the given triangle is a .

In a right angled triangle the area of the triangle ‘A’ is given by,

In a right angled triangle the sides containing the right angle will not be the longest side.

Hence the area of the given right angled triangle is,

Hence the area of the given right-angled triangle is.

In a right-angled triangle the hypotenuse will be the longest side. Here the longest side is ‘BC’.

Hence the hypotenuse of the given right-angled triangle is

Page No 277:

Question 9:

Prove that the points (2,3), (−4, −6) and (1, 3/2) do not form a triangle.

Answer:

The distance d between two points and is given by the formula

In any triangle the sum of lengths of any two sides need to be greater than the third side.

Here the three points are, and

Let us now find out the lengths of all the three sides of the given triangle.

Here we see that,

This is in violation of the basic property of any triangle to exist. Therefore these points cannot give rise to a triangle.

Hence we have proved that the given three points do not form a triangle.

Page No 277:

Question 10:

The points A(2, 9), B(a, 5) and C(5, 5) are the vertices of a triangle ABC right angled at B. Find the values of a and hence the area of ΔABC.

Answer:


Consider the figure. 
Using distance formula, 
AC=2-52+9-52AC=-32+42AC=25AC2=25 units
AB=2-a2+9-52AB=4+a2-4a+16AB=4+a2-4a+16=a2-4a+20AB2=(a2-4a+20) units
BC=5-a2+5-52BC=25+a2-10a+0BC=a2-10a+25BC2=(a2-10a+25) units
We are given that ABC is a right triangle right angled at B. 
By Pythagoras theorem, we have;
AC2=AB2+BC225=(a2-4a+20)+(a2-10a+25)25=2a2-14a+452a2-14a+20=0a2-7a+10=0a2-5a-2a+10=0a(a-5)-2(a-5)=0(a-2)(a-5)=0a=2 or a=5
We cannot put a = 5 as it will make BC = 0. So, we ignore a = 5 and accept a = 2.
AB2=4-8+20=16AB=4 unitsBC2=4+25-20=9BC=3 unitsAnd, AC=5 unitsArea=12×base×heightArea=12×3×4=6 square units
 

Page No 277:

Question 11:

If the point P(2, 2) is equidistant from the points A(−2, k) and B(−2k, −3), find k. Also, find the length of AP.                     [CBSE 2014]

Answer:

It is given that P(2, 2) is equidistant from the points A(−2, k) and B(−2k, −3).

∴ AP = BP

-2-22+k-22=-2k-22+-3-22                (Distance formula)

Squaring on both sides, we get

16+k2-4k+4=4k2+8k+4+253k2+12k+9=0k2+4k+3=0k+3k+1=0
k+3=0 or k+1=0k=-3 or k=-1

Thus, the value of k is −1 or −3.

When k = −1,

AP=-2-22+-1-22=-42+-32=16+9=25=5 units

When k = −3,

AP=-2-22+-3-22=-42+-52=16+25=41 units

Page No 277:

Question 12:

Find a relation between x and y such that the point (x, y) is equidistant from the points (3, 6) and (−3, 4).

Answer:

The distance d between two points and is given by the formula

Let the three given points be P(x, y), A(3,6) and B(3,4).

Now let us find the distance between ‘P’ and ‘A’.

Now, let us find the distance between ‘P’ and ‘B’.

It is given that both these distances are equal. So, let us equate both the above equations,

Squaring on both sides of the equation we get,

Hence the relationship between ‘x’ and ‘y’ based on the given condition is.

Page No 277:

Question 13:

Prove that the points (−2, 5), (0, 1) and (2, −3)  are collinear.

Answer:

The distance d between two points and is given by the formula

For three points to be collinear the sum of distances between two pairs of points should be equal to the third pair of points.

The given points are A (2, 5), B (0, 1) and C (2, 3)

Let us find the distances between the possible pairs of points.

We see that

Since sum of distances between two pairs of points equals the distance between the third pair of points the three points must be collinear.

Hence we have proved that the three given points are.

Page No 277:

Question 14:

If the point A(2, –4) is equidistant from P(3, 8) and Q(–10, y), find the value of y. Also find the distance PQ.

Answer:

The distance between two points x1,y1 and x2,y2 is given as:
d=x2-x12+y2-y12

Now, the point A(2, –4) is equidistant from P(3, 8) and Q(–10, y).
Thus,
3-22+8+42=2+102+-4-y21+144=144+16+y2+8y145=y2+8y+160

Squaring both sides,
145=y2+8y+160y2+8y+15=0y2+5y+3y+15=0yy+5+3y+5=0y+3y+5=0y=-3 or y=-5

Thus, the value of y is either -3 or -5. Therefore, Q(–10, –3) or Q(–10, –5). 

Now, the distance PQ if Q(–10, –3) is given as:
PQ=3+102+8+32=169+121=290

Similarly, the distance PQ if Q(–10, –5) is given as:
PQ=3+102+8+52=169+169=238=132

Therefore, the distance PQ is either 290 units or 132 units.

Page No 277:

Question 15:

The three vertices of a parallelogram are (3, 4) (3, 8) and (9, 8). Find the fourth vertex.

Answer:

We are given three vertices of a parallelogram A(3, 4),  B(3, 8) and C(9, 8). 

We know that diagonals of a parallelogram bisect each other. Let the fourth vertex be D(x, y).
Mid point of BD = x+32, y+82
Mid point of AC = 9+32, 4+82 = 6, 6
Since the mid point of BD = mid point of AC
So, x+32, y+82 = (6, 6)
Thus, x = 9, y = 4.
So, the fourth vertex is (9, 4).

 

Page No 277:

Question 16:

Name the quadrilateral formed, if any, by the following points, and give reasons for your answers:

(i) A(2, –2), B(7, 3), C(1, –1), D(6, –6)

(ii) A(4, 5), B(7, 6), C(4, 3), D(1, 2)

Answer:

Disclaimer: In (i), the coordinates of point C should be (11, –1).

(i) A(2, –2), B(7, 3), C(11, –1), D(6, –6)

Now, using the distance formula we can find the sides of the quadrilateral.

AB=7-22+3+22=25+25=50=52
BC=11-72+-1-32=42+-42=32=42
CD=6-112+-6+12=50=52
AD=2-62+-2+62=32=42

Similarly, we can find the length of diagonals.
AC=11-22+-1+22=81+1=82
BD=6-72+-6-32=1+81=82

Thus, we conclude AB = CD and BC = AD.
Also, AC = BD, diagonals are equal.
Hence, the required quadrilateral is a rectangle.

(ii) A(4, 5), B(7, 6), C(4, 3), D(1, 2)

The distance between two points Pand Qis given by distance formula:



Thus,
AB=7-42+6-52=9+1=10

BC=4-72+3-62=9+9=18

CD=1-42+2-32=9+1=10

AD=4-12+5-22=9+9=18

Hence,

AB = CD and BC = DA

Here, opposite sides of the quadrilateral is equal.

Now, 
AC=4-42+3-52=4=2

BD=7-12+6-22=36+16=52

Since the diagonals are not equal, it is a parallelogram.

Page No 277:

Question 17:

Prove that the points (3, 0), (4, 5), (−1, 4) and (−2 −1), taken in order, form a rhombus. Also, find its area.

Answer:

The distance d between two points and is given by the formula

In a rhombus all the sides are equal in length. And the area ‘A’ of a rhombus is given as

Here the four points are A(3,0), B(4,5), C(1,4) and D(2,1).

First let us check if all the four sides are equal.

Here, we see that all the sides are equal, so it has to be a rhombus.

Hence we have proved that the quadrilateral formed by the given four vertices is a.

Now let us find out the lengths of the diagonals of the rhombus.

Now using these values in the formula for the area of a rhombus we have,

Thus the area of the given rhombus is.

Page No 277:

Question 18:

In the seating arrangement of desks in a classroom three students Rohini, Sandhya and Bina are seated at A(3, 1), B(6, 4), and C(8, 6). Do you think they are seated in a line?

Answer:

The distance d between two points and is given by the formula

For three points to be collinear the sum of distances between any two pairs of points should be equal to the third pair of points.

The given points are A(3,1), B(6,4) and C(8,6).

Let us find the distances between the possible pairs of points.

We see that.

Since sum of distances between two pairs of points equals the distance between the third pair of points the three points must be collinear.

Hence, the three given points are.

Page No 277:

Question 19:

Find a point on y-axis which is equidistant form the points (5, −2) and (−3, 2).

Answer:

The distance d between two points and is given by the formula

Here we are to find out a point on the y−axis which is equidistant from both the points A(5,2) and B(3,2).

Let this point be denoted as C(x, y).

Since the point lies on the y-axis the value of its ordinate will be 0. Or in other words we have.

Now let us find out the distances from ‘A’ and ‘B’ to ‘C

We know that both these distances are the same. So equating both these we get,

Squaring on both sides we have,

Hence the point on the y-axis which lies at equal distances from the mentioned points is.

Page No 277:

Question 20:

Find a point on the x-axis which is equidistant from the points (7, 6) and (−3, 4).

Answer:

The distance d between two points and is given by the formula

Here we are to find out a point on the x−axis which is equidistant from both the points A(7,6) and B(3,4).

Let this point be denoted as C(x, y).

Since the point lies on the x-axis the value of its ordinate will be 0. Or in other words we have.

Now let us find out the distances from ‘A’ and ‘B’ to ‘C

We know that both these distances are the same. So equating both these we get,

Squaring on both sides we have,

Hence the point on the x-axis which lies at equal distances from the mentioned points is.

Page No 277:

Question 21:

(i) Prove that the points A (2, 3), B(−2, 2), C(−1,−2) and D(3, −1) are the vertices of a square ABCD.

(ii) Name the type of triangle PQR formed by the points P2, 2, Q-2, 2 and R-6, 6 

Answer:

(i) A (2, 3), B(−2, 2), C(−1,−2) and D(3, −1)

Now, using the distance formula we can find the sides of the quadrilateral.

AB=-2-22+2-32=16+1=17
BC=-1+22+-2-22=12+-42=17
CD=3+12+-1+22=17
AD=3-22+-1-32=1+16=17

Similarly, we can find the length of diagonals.
AC=-1-22+-2-32=9+25=34
BD=3+22+-1-22=25+9=34

Thus, we conclude AB = BC = CD = AD.
Also, AC = BD, diagonals are equal.
Hence, the required quadrilateral is a square.

(ii) P2, 2, Q-2, 2 and R-6, 6
Now, using the distance formula we can find the sides of the triangle.

PQ=-2-22+-2-22=8+8=16=4

QR=-6+22+6+22=6+2-212+6+2+212=16=4

PR=-6-22+6-22=6+2+212+6+2-212=16=4

We conclude PQ = QR = PR.
Hence, the required triangle is equilateral.

Page No 277:

Question 22:

Find the value of x such that PQ = QR where the coordinates of P, Q and R are (6, −1) , (1, 3) and (x, 8) respectively.

Answer:

The distance d between two points and is given by the formula

The three given points are P(6,1), Q(1,3) and R(x,8).

Now let us find the distance between ‘P’ and ‘Q’.

Now, let us find the distance between ‘Q’ and ‘R’.

It is given that both these distances are equal. So, let us equate both the above equations,

Squaring on both sides of the equation we get,

Now we have a quadratic equation. Solving for the roots of the equation we have,

Thus the roots of the above equation are 5 and −3.

Hence the values of ‘x’ are.

Page No 277:

Question 23:

If Q (0,1) is equidistant from P (5, −3) and R (x, 6), find the values of x. Also, find the distances QR and PR.

Answer:

The distance d between two points and is given by the formula

The three given points are Q(0,1), P(5,3) and R(x,6).

Now let us find the distance between ‘P’ and ‘Q’.

Now, let us find the distance between ‘Q’ and ‘R’.

It is given that both these distances are equal. So, let us equate both the above equations,

Squaring on both sides of the equation we get,

Hence the values of ‘x’ are.

Now, the required individual distances,

Hence the length of ‘QR’ is.

For ‘PR’ there are two cases. First when the value of ‘x’ is 4,

Then when the value of ‘x’ is −4,

Hence the length of ‘PR’ can beunits



Page No 278:

Question 24:

Find the values of y for which the distance between the points P (2, −3) and Q(10,y) is 10 units.

Answer:

The distance d between two points and is given by the formula

The distance between two points P(2,3) and Q(10,y) is given as 10 units. Substituting these values in the formula for distance between two points we have,

Now, squaring the above equation on both sides of the equals sign

Thus we arrive at a quadratic equation. Let us solve this now,

The roots of the above quadratic equation are thus 3 and −9.

Thus the value of ‘y’ could either be.

Page No 278:

Question 25:

If A(3, y) is equidistant from points P(8, −3) and Q(7, 6), find the value of y and find the distance AQ.                      [CBSE 2014]

Answer:

It is given that A(3, y) is equidistant from points P(8, −3) and Q(7, 6).

∴ AP = AQ

3-82+y--32=3-72+y-62                 (Distance formula)

Squaring on both sides, we get

25+y+32=16+y-6225+y2+6y+9=16+y2-12y+3612y+6y=52-3418y=18y=1

Thus, the value of y is 1.

AQ=3-72+1-62=-42+-52=16+25=41 units

Page No 278:

Question 26:

Find the equation of the perpendicular bisector of the line segment joining points (7, 1) and (3,5).

Answer:

TO FIND: The equation of perpendicular bisector of line segment joining points (7, 1) and (3, 5)

Let P(x, y) be any point on the perpendicular bisector of AB. Then,

PA=PB

Hence the equation of perpendicular bisector of line segment joining points (7, 1) and (3, 5) is

Page No 278:

Question 27:

The centre of a circle is (2a, a-7). Find the values of a if the circle passes through the point (11,-9) and has diameter 102 units.

Answer:

The length of the diameter is 102 units.
So, the radius is 52 units.
The centre of the circle be C(2a, a−7).
Suppose it passes through the point P(11, −9).
Therefore, PC = r
 PC2=r211-2a2+-9-a+72=522121+4a2-44a+a2+4+4a=505a2-40a+75=0a-3a-5=0a=3 or a=5
Hence the values of a are 3 or 5.
 

Page No 278:

Question 28:

Ayush starts walking from his house to office . Instead of going to the office directly , he goes to a bank first , from there to his daughter 's school and then reaches the office. what is the extra distance travelled by Ayush in reaching the office ? ( Assume that all distances covered are in straight lines) . If the house is situated at (2,4) , bank at (5,8) , school at (13,14)  and office at ( 13,26)  and coordinates are in kilometers .

Answer:

The position of the ayush's house is (2, 4) and that of the bank is (5, 8).
The distance between the house and the bank is
d1=5-22+8-42=32+42=9+16=5 units
The position of the the bank is (5, 8) and that of the school is (13, 14).
The distance between the bank and the school is
d2=13-52+14-82=82+62=64+36=10 units
The position of the school is (13, 14) and that of the office is (13, 6)
The distance between the bank and the school is
d3=13-132+26-142=02+122=12 units
Suppose d be the total distance travelled by ayush
d=d1+d2+d3d=5+10+12d=27 km
Now, let D be the shortest distance between ayush house and the office,
D=13-22+26-42=112+222=605 =24.6 km

Thus the extra distance covered by ayush is d − D = 27 − 24.6 = 2.4 km.

Page No 278:

Question 29:

If (0, −3) and (0, 3) are the two vertices of an equilateral triangle, find the coordinates of its third vertex.              [CBSE 2014]

Answer:

Let the given points be A(0, −3) and B(0, 3). Suppose the coordinates of the third vertex be C(x, y).

Now, ∆ABC is an equilateral triangle.

∴ AB = BC = CA

0-02+-3-32=x-02+y-32=x-02+y--32                 (Distance formula)

Squaring on both sides, we get

36=x2+y-32=x2+y+32

x2+y-32=x2+y+32 and x2+y-32=36

Now,

x2+y-32=x2+y+32y2-6y+9=y2+6y+9-12y=0y=0

Putting y = 0 in x2+y-32=36, we get

x2+0-32=36x2=36-9=27x=±27=±33

Thus, the coordinates of the third vertex are 33, 0 or -33, 0.

Page No 278:

Question 30:

An equilateral triangle has two vertices at the points (3, 4) and (−2, 3), find the coordinates of the third vertex.

Answer:

The distance d between two points and is given by the formula

In an equilateral triangle all the sides are of equal length.

Here we are given that A (3, 4) and B (2, 3) are two vertices of an equilateral triangle. Let C(x, y) be the third vertex of the equilateral triangle.

First let us find out the length of the side of the equilateral triangle.

Hence the side of the equilateral triangle measures units.

Now, since it is an equilateral triangle, all the sides need to measure the same length.

Hence we have

Equating both these equations we have,

Squaring on both sides we have,

From the above equation we have,

Substituting this and the value of the side of the triangle in the equation for one of the sides we have,

Squaring on both sides,

Now we have a quadratic equation for ‘x’. Solving for the roots of this equation,

We know that. Substituting the value of ‘x’ we have,

Hence the two possible values of the third vertex are.

Page No 278:

Question 31:

Find the circumcentre of the triangle whose vertices are (−2, −3), (−1, 0), (7, −6).

Answer:

The distance d between two points and is given by the formula

The circumcentre of a triangle is the point which is equidistant from each of the three vertices of the triangle.

Here the three vertices of the triangle are given to be A(2.−3), B(1,0) and C(7,6).

Let the circumcentre of the triangle be represented by the point R(x, y).

So we have

Equating the first pair of these equations we have,

Squaring on both sides of the equation we have,

Equating another pair of the equations we have,

Squaring on both sides of the equation we have,

Now we have two equations for ‘x’ and ‘y’, which are

From the second equation we have. Substituting this value of ‘y’ in the first equation we have,

Therefore the value of ‘y’ is,

Hence the co−ordinates of the circumcentre of the triangle with the given vertices are.

Page No 278:

Question 32:

Find the angle subtended at the origin by the line segment whose end points are (0, 100) and (10, 0).

Answer:

The distance d between two points and is given by the formula

In a right angled triangle the angle opposite the hypotenuse subtends an angle of.

Here let the given points be A(0,100), B(10,0). Let the origin be denoted by O(0,0).

Let us find the distance between all the pairs of points

Here we can see that.

So, is a right angled triangle with ‘AB’ being the hypotenuse. So the angle opposite it has to be. This angle is nothing but the angle subtended by the line segment ‘AB’ at the origin.

Hence the angle subtended at the origin by the given line segment is.

Page No 278:

Question 33:

Two opposite vertices of a square are (−1, 2) and (3, 2). Find the coordinates of other two vertices.

Answer:

The distance d between two points and is given by the formula

In a square all the sides are of equal length. The diagonals are also equal to each other. Also in a square the diagonal is equal to times the side of the square.

Here let the two points which are said to be the opposite vertices of a diagonal of a square be A(1,2) and C(3,2).

Let us find the distance between them which is the length of the diagonal of the square.

Now we know that in a square,

Substituting the value of the diagonal we found out earlier in this equation we have,

Now, a vertex of a square has to be at equal distances from each of its adjacent vertices.

Let P(x, y) represent another vertex of the same square adjacent to both ‘A’ and ‘C’.

But these two are nothing but the sides of the square and need to be equal to each other.

Squaring on both sides we have,

Substituting this value of ‘x’ and the length of the side in the equation for ‘AP’ we have,

Squaring on both sides,

We have a quadratic equation. Solving for the roots of the equation we have,

The roots of this equation are 0 and 4.

Therefore the other two vertices of the square are.



Page No 286:

Question 1:

Find the coordinates of the point which divides the line segment joining (−1,3) and (4, −7) internally in the ratio 3 : 4

Answer:

We have A (−1, 3) and B (4,−7) be two points. Let a pointdivide the line segment joining the points A and B in the ratio 3:4 internally.

Now according to the section formula if point a point P divides a line segment joining andin the ratio m: n internally than,

Now we will use section formula to find the co-ordinates of unknown point P as,

Therefore, co-ordinates of point P is

Page No 286:

Question 2:

Find the coordinates of the point where the diagonals of the parallelogram formed by joining the points (−2, −1), (1, 0), (4, 3) and(1, 2) meet.

Answer:

The co-ordinates of the midpoint between two points and is given by,

In a parallelogram the diagonals bisect each other. That is the point of intersection of the diagonals is the midpoint of either of the diagonals.

Here, it is given that the vertices of a parallelogram are A(−2,−1), B(1,0) and C(4,3) and D(1,2).

We see that ‘AC’ and ‘BD’ are the diagonals of the parallelogram.

The midpoint of either one of these diagonals will give us the point of intersection of the diagonals.

Let this point be M(x, y).

Let us find the midpoint of the diagonal ‘AC’.

Hence the co-ordinates of the point of intersection of the diagonals of the given parallelogram are.

Page No 286:

Question 3:

Point A(3, 1), B(5, 1), C(a, b) and D(4, 3) are vertices of a parallelogram ABCD. Find the values of a and b.

Answer:

Given: Point A(3, 1), B(5, 1), C(a, b) and D(4, 3) are vertices of a parallelogram ABCD.

Diagonals of a parallelogram bisect each other.

∴ Mid point of AC = Mid point of BD

Mid point of x1, y1 and x2, y2 is x1+x22, y1+y22.Mid point of AC=3+a2, 1+b2Mid point of BD=5+42, 1+32                           =92, 42=92, 23+a2, 1+b2=92, 23+a2=92 and 1+b2=23+a=9 and 1+b=4a=6 and b=3

Hence,  the values of a and is 6 and 3, respectively.

Page No 286:

Question 4:

Find the ratio in which the point (2, y) divides the line segment joining the points A (−2, 2) and B (3, 7). Also, find the value of y.

Answer:

The co-ordinates of a point which divided two points and internally in the ratio is given by the formula,

Here we are given that the point P(2,y) divides the line joining the points A(−2,2) and B(3,7) in some ratio.

Let us substitute these values in the earlier mentioned formula.

Equating the individual components we have

We see that the ratio in which the given point divides the line segment is.

Let us now use this ratio to find out the value of ‘y’.

Equating the individual components we have

Thus the value of ‘y’ is.

Page No 286:

Question 5:

If A (−1, 3), B (1, −1) and C (5, 1) are the vertices of a triangle ABC, find the length of the median through A.

Answer:

The distance d between two points and is given by the formula

The co-ordinates of the midpoint between two points and is given by,

Here, it is given that the three vertices of a triangle are A(−1,3), B(1,−1) and C(5,1).

The median of a triangle is the line joining a vertex of a triangle to the mid-point of the side opposite this vertex.

Let ‘D’ be the mid-point of the side ‘BC’.

Let us now find its co-ordinates.

Thus we have the co-ordinates of the point as D(3,0).

Now, let us find the length of the median ‘AD’.

Thus the length of the median through the vertex ‘A’ of the given triangle is.



Page No 287:

Question 6:

The points (3, −4) and (−6, 2) are the extremities of a diagonal of a parallelogram. If the third vertex is (−1,−3). Find the coordinates of the fourth vertex.

Answer:

Let ABCD be a parallelogram in which the co-ordinates of the vertices are A (3,−4); B (−1,−3) and C (−6, 2). We have to find the co-ordinates of the forth vertex.

Let the forth vertex be

Since ABCD is a parallelogram, the diagonals bisect each other. Therefore the mid-point of the diagonals of the parallelogram will coincide.

Now to find the mid-point of two pointsand we use section formula as,

The mid-point of the diagonals of the parallelogram will coincide.

So,

Therefore,

Now equate the individual terms to get the unknown value. So,

So the forth vertex is

Page No 287:

Question 7:

Find the coordinates of the points which divide the line segment joining A(−2, 2) and B (2, 8) into four equal parts.

Answer:

The co-ordinates of the midpoint between two points and is given by,

Here we are supposed to find the points which divide the line joining A(−2,2) and B(2,8) into 4 equal parts.

We shall first find the midpoint M(x, y) of these two points since this point will divide the line into two equal parts.

So the point M(0,5) splits this line into two equal parts.

Now, we need to find the midpoint of A(−2,2) and M(0,5) separately and the midpoint of B(2,8) and M(0,5). These two points along with M(0,5) split the line joining the original two points into four equal parts.

Let be the midpoint of A(−2,2) and M(0,5).

Now let bet the midpoint of B(2,8) and M(0,5).

Hence the co-ordinates of the points which divide the line joining the two given points are.

Page No 287:

Question 8:

Points P, Q, R and S divides the line segment joining A(1, 2) and B(6, 7) in 5 equal parts. Find the coordinates of the points P, Q and R.                                                                                                                                                                                          [CBSE 2014]

Answer:



It is given that P, Q, R and S divides the line segment joining A(1, 2) and B(6, 7) in 5 equal parts.

∴ AP = PQ = QR = RS = SB          .....(1)

Now,

AP + PQ + QR + RS + SB = AB

⇒ AP + AP + AP + AP + AP = AB             [From (1)]

⇒ 5AP = AB

⇒ AP = 15AB                  .....(2)   

Now,

PB = PQ + QR + RS + SB = 15AB +15AB + 15AB + 15AB = 45AB         .....(3)

From (2) and (3), we get

AP : PB = 15AB : 45AB = 1 : 4

Similarly,

AQ : QB = 2 : 3 and AR : RB = 3 : 2

Using section formula, we get

Coordinates of P = 1×6+4×11+4,1×7+4×21+4=105,155=2,3

Coordinates of Q = 2×6+3×12+3,2×7+3×22+3=155,205=3,4

Coordinates of R = 3×6+2×13+2,3×7+2×23+2=205,255=4,5

Page No 287:

Question 9:

Find the ratio in which the line segment joining (−2, −3) and (5, 6) is divided by (i) x-axis (ii) y-axis. Also, find the coordinates of the point of division in each case.

Answer:

The ratio in which the x−axis divides two points  and  is λ:1

The ratio in which the y-axis divides two points  and  is μ:1

The co-ordinates of the point dividing two points  and  in the ratio  is given as,

 Where 

Here the two given points are A(−2,−3) and B(5,6).

  1. The ratio in which the x-axis divides these points is

6λ-33=0λ=12

Let point P(x, y) divide the line joining ‘AB’ in the ratio 

Substituting these values in the earlier mentioned formula we have,

Thus the ratio in which the x−axis divides the two given points and the co-ordinates of the point is.

  1. The ratio in which the y-axis divides these points is

5μ-23=0μ=25

Let point P(x, y) divide the line joining ‘AB’ in the ratio 

Substituting these values in the earlier mentioned formula we have,

Thus the ratio in which the x-axis divides the two given points and the co-ordinates of the point is.

Page No 287:

Question 10:

Prove that (4, 3), (6, 4) (5, 6) and (3, 5)  are the angular points of a square.

Answer:

Let A (4, 3); B (6, 4); C (5, 6) and D (3, 5) be the vertices of a quadrilateral. We have to prove that the quadrilateral ABCD is a square.

So we should find the lengths of sides of quadrilateral ABCD.

All the sides of quadrilateral are equal.

So now we will check the lengths of the diagonals.

All the sides as well as the diagonals are equal. Hence ABCD is a square.

Page No 287:

Question 11:

Prove that the points (−4,−1), (−2, 4), (4, 0) and (2, 3) are the vertices of a rectangle.

Answer:

Let A (−4,−1); B (−2,−4); C (4, 0) and D (2, 3) be the vertices of a quadrilateral. We have to prove that the quadrilateral ABCD is a rectangle.

So we should find the lengths of opposite sides of quadrilateral ABCD.

Opposite sides are equal. So now we will check the lengths of the diagonals.

Opposite sides are equal as well as the diagonals are equal. Hence ABCD is a rectangle.

Page No 287:

Question 12:

ABCD is a rectangle formed by joining the points A (−1, −1), B(−1 4) C (5 4) and D (5, −1). P, Q, R and S are the mid-points of sides AB, BC, CD and DA respectively. Is the quadrilateral PQRS a square? a rectangle? or a rhombus? Justify your answer.

Answer:

We have a rectangle ABCD formed by joining the points A (−1,−1); B (−1, 4); C (5, 4) and D (5,−1). The mid-points of the sides AB, BC, CD and DA are P, Q, R, S respectively.

We have to find that whether PQRS is a square, rectangle or rhombus.

In general to find the mid-point of two pointsand we use section formula as,

Therefore mid-point P of side AB can be written as,

Now equate the individual terms to get,

So co-ordinates of P is

Similarly mid-point Q of side BC can be written as,

Now equate the individual terms to get,

So co-ordinates of Q is (2, 4)

Similarly mid-point R of side CD can be written as,

Now equate the individual terms to get,

So co-ordinates of R is

Similarly mid-point S of side DA can be written as,

Now equate the individual terms to get,

So co-ordinates of S is (2,−1)

So we should find the lengths of sides of quadrilateral PQRS.

All the sides of quadrilateral are equal.

So now we will check the lengths of the diagonals.

All the sides are equal but the diagonals are unequal. Hence ABCD is a rhombus.

Page No 287:

Question 13:

Find the ratio in which the point P(−1, y) lying on the line segment joining A(−3, 10) and B(6 −8) divides it. Also find the value of y.                                                                                                                                                                                            [CBSE 2013]

Answer:

Suppose P(−1, y) divides the line segment joining A(−3, 10) and B(6 −8) in the ratio k : 1.

Using section formula, we get

Coordinates of P = 6k-3k+1,-8k+10k+1

6k-3k+1,-8k+10k+1=-1,y

6k-3k+1=-1 and y=-8k+10k+1

Now,

6k-3k+1=-16k-3=-k-17k=2k=27

So, P divides the line segment AB in the ratio 2 : 7.

Putting k = 27 in y=-8k+10k+1, we get

y=-8×27+1027+1=-16+702+7=549=6

Hence, the value of y is 6.

Page No 287:

Question 14:

Find the coordinates of a point A, where AB is a diameter of the circle whose centre is (2, −3) and B is (1, 4).

Answer:

Let the co-ordinates of point A be.

Centre lies on the mid-point of the diameter. So applying the mid-point formula we get,

Similarly,

So the co-ordinates of A are (3,−10)

Page No 287:

Question 15:

In what ratio does the point (−4, 6) divide the line segment joining the points A(−6, 10) and B(3,−8)?

Answer:

The co-ordinates of a point which divided two points and internally in the ratio is given by the formula,

Here it is said that the point (−4,6) divides the points A(−6,10) and B(3,−8). Substituting these values in the above formula we have,

Equating the individual components we have,

Therefore the ratio in which the line is divided is

Page No 287:

Question 16:

Find the ratio in which the y-axis divides the line segment joining the points (5, −6)  and (−1,−4). Also, find the coordinates of the point of division.

Answer:

The ratio in which the y-axis divides two points and is λ:1

The co-ordinates of the point dividing two points and in the ratio is given as,

where,

Here the two given points are A(5,−6) and B(−1,−4).

(x, y)=-λ+5λ+1,-4λ-6λ+1
Since, the y-axis divided the given line, so the x coordinate will be 0.
-λ+5λ+1=0λ=51

Thus the given points are divided by the y-axis in the ratio.

The co-ordinates of this point (x, y) can be found by using the earlier mentioned formula.

Thus the co-ordinates of the point which divides the given points in the required ratio are.

Page No 287:

Question 17:

If A and B are (1, 4) and (5, 2) respectively, find the coordinates of P when AP/BP = 3/4.

Answer:

The co-ordinates of the point dividing two points and in the ratio is given as,

where,

Here the two given points are A(1,4) and B(5,2). Let point P(x, y) divide the line joining ‘AB’ in the ratio

Substituting these values in the earlier mentioned formula we have,

Thus the co-ordinates of the point which divides the given points in the required ratio are.

Page No 287:

Question 18:

Find the ratio in which P(4, m) divides the line segment joining the points A(2, 3) and B(6, –3). Hence, find m.

Answer:

Let P divides AB in a ratio of λ : 1
Therefore, coordinates of the point P are 6λ+2λ+1, -3λ+3λ+1
Given that coordinates of the point P are (4, m).
6λ+2λ+1=46λ+2=4λ+4λ=1
Hence, the point P divides AB in a ratio of 1 : 1.
Replacing the value of λ = 1 in y-coordinate of P, we get
-31+31+1=mm=0
Thus, y-coordinate of P is equal to 0. 

Page No 287:

Question 19:

Find the coordinate of the point R on the line segment joining the points P(–1, 3) and Q(2, 5) such that PR=35PQ.

Answer:

Given that, point R lies on the line segment PQ where P(–1, 3) and Q(2, 5) and PR=35PQ. This means that point R divides PQ in the ratio 3 : 2 internally.

If a point X(x, y) divides AB with A(x1, y1) and B(x2, y2) in a ratio of m : n, then coordinates of the point X are mx2+nx1m+n,my2+ny1m+n.

Thus,
Coordinates of the point R = 3×2+2×-13+2,3×5+2×33+2
                                           =6-25,15+65=45,215

Hence, the coordinates of the point R are 45,215.

Page No 287:

Question 20:

​If (a, b) is the mid-point of the line segment joining the points A(10, –6), B(k, 4) and a – 2= 18, find the value of and the distance AB.

Answer:

Let the point X(a, b) be the mid-point of the line segment joining the points A(10, –6) and B(k, 4).

Now, coordinates of the midpoint of a line segment with end points (x1, y1) and (x2, y2) are given as x1+x22,y1+y22.

Thus,
 a,b=10+k2,-6+42b=-1 and a=10+k2

Given that, a – 2= 18.
Putting the respective values in the given equation,
10+k2-2-1=1810+k2+2=1810+k2=16k=22

Thus, k = 22.

Now, the distance between the points A(10, –6) and B(22, 4) is given as:
AB=22-102+4+62                               d=x2-x12+y2-y12=122+102=144+100=244=261

Hence, the distance AB is 261 units.

Page No 287:

Question 21:

If the points P, Q(x, 7), R, S(6, y) in this order divide the line segment joining A(2, p) and B(7, 10) in 5 equal parts, find x, y and p.                                                                                                                                                                                                         [CBSE 2015]

Answer:



It is given that P, Q(x, 7), R, S(6, y) divides the line segment joining A(2, p) and B(7, 10) in 5 equal parts.

∴ AP = PQ = QR = RS = SB          .....(1)

Now,

AP + PQ + QR + RS + SB = AB

⇒ SB + SB + SB + SB + SB = AB             [From (1)]

⇒ 5SB = AB

⇒ SB = 15AB                  .....(2)   

Now,

AS = AP + PQ + QR + RS = 15AB +15AB + 15AB + 15AB = 45AB         .....(3)

From (2) and (3), we get

AS : SB = 45AB : 15AB = 4 : 1

Similarly,

AQ : QB = 2 : 3

Using section formula, we get

Coordinates of Q = 2×7+3×22+3,2×10+3×p2+3=205,20+3p5=4,20+3p5

x,7=4,20+3p5

x=4 and 7=20+3p5

Now,

7=20+3p520+3p=353p=15p=5

Coordinates of S = 4×7+1×24+1,4×10+1×p4+1=305,40+55=6,9

6,y=6,9y=9

Thus, the values of x, y and p are 4, 9 and 5, respectively.

Page No 287:

Question 22:

If a vertex of a triangle be (1, 1) and the middle points of the sides through it be (−2,−3) and (5 2) find the other vertices.

Answer:

Let a in which P and Q are the mid-points of sides AB and AC respectively. The coordinates are: A (1, 1); P (−2, 3) and Q (5, 2).

We have to find the co-ordinates of and.

In general to find the mid-point of two pointsand we use section formula as,

Therefore mid-point P of side AB can be written as,

Now equate the individual terms to get,

So, co-ordinates of B is (−5, 5)

Similarly, mid-point Q of side AC can be written as,

 

Now equate the individual terms to get,

So, co-ordinates of C is (9, 3)

Page No 287:

Question 23:

If the mid-point of the line joining (3, 4) and (k, 7) is (x, y) and 2x + 2y + 1 = 0 find the value of k.

Answer:

We have two points A (3, 4) and B (k, 7) such that its mid-point is.

It is also given that point P lies on a line whose equation is

In general to find the mid-point of two pointsand we use section formula as,

Therefore mid-point P of side AB can be written as,

Now equate the individual terms to get,

Since, P lies on the given line. So,

Put the values of co-ordinates of point P in the equation of line to get,

On further simplification we get,

So,

Page No 287:

Question 24:

If A and B are two points having coordinates (−2, −2) and (2, −4) respectively, find the coordinates of P such that AP = 37AB.

Answer:

We have two points A (−2,−2) and B (2,−4). Let P be any point which divide AB as,

Since,

So,

Now according to the section formula if any point P divides a line segment joining andin the ratio m: n internally than,

Therefore P divides AB in the ratio 3: 4. So,

Page No 287:

Question 25:

If two vertices of a parallelogram are (3, 2) (−1, 0) and the diagonals cut at (2, −5), find the other vertices of the parallelogram.

Answer:

We have a parallelogram ABCD in which A (3, 2) and B (−1, 0) and the co-ordinate of the intersection of diagonals is M (2,−5).

We have to find the co-ordinates of vertices C and D.

So let the co-ordinates be and

In general to find the mid-point of two pointsand we use section formula as,

The mid-point of the diagonals of the parallelogram will coincide.

So,

Therefore,

Now equate the individual terms to get the unknown value. So,

So the co-ordinate of vertex C is (1,−12)

Similarly,

Therefore,

Now equate the individual terms to get the unknown value. So,

So the co-ordinate of vertex C is (5,−10)

Page No 287:

Question 26:

If the coordinates of the mid-points of the sides of a triangle are (3, 4) (4, 6) and (5, 7), find its vertices.

Answer:

The co-ordinates of the midpoint between two points and is given by,

Let the three vertices of the triangle be, and.

The three midpoints are given. Let these points be, and.

Let us now equate these points using the earlier mentioned formula,

Equating the individual components we get,

Using the midpoint of another side we have,

Equating the individual components we get,

Using the midpoint of the last side we have,

Equating the individual components we get,

Adding up all the three equations which have variable ‘x’ alone we have,

Substituting in the above equation we have,

Therefore,

And

Adding up all the three equations which have variable ‘y’ alone we have,

Substituting in the above equation we have,

Therefore,

And

Therefore the co-ordinates of the three vertices of the triangle are.

Page No 287:

Question 27:

The line segment joining the points P(3, 3) and Q(6, −6) is trisected at the points A and B such that A is nearer to P. If A also lies on the line given by 2x + y + k = 0, find the value of k.

Answer:

We have two points P (3, 3) and Q (6,−6). There are two points A and B which trisect the line segment joining P and Q.

Let the co-ordinate of A be

Now according to the section formula if any point P divides a line segment joining andin the ratio m: n internally than,

The point A is the point of trisection of the line segment PQ. So, A divides PQ in the ratio 1: 2

Now we will use section formula to find the co-ordinates of unknown point A as,

Therefore, co-ordinates of point A is(4, 0)

It is given that point A lies on the line whose equation is

So point A will satisfy this equation.

So,



Page No 288:

Question 28:

The line segment joining the points (3, −4) and (1, 2) is trisected at the points P and Q. If the coordinates of P and Q are (p, −2) and (5/3, q) respectively. Find the values of p and q.

Answer:

Let the points A(3, −4) and B(1, 2) is trisected at the points P(p, −2) and Q(5/3, q).

Thus, AP = PQ = QB.

Therefore, P divides AB internally in the ratio 1 : 2. 

Section formula: if the point (xy) divides the line segment joining the points (x1y1) and (x2y2) internally in the ratio : n, then the coordinates (xy) = mx2+nx1m+n, my2+ny1m+n

Therefore, using section formula, the coordinates of P are:

p, -2=11+231+2,12+2-41+2p, -2=1+63,2-83p, -2=73,-63p, -2=73,-2p=73

Also, Q divides AB internally in the ratio 2 : 1.

Therefore, using section formula, the coordinates of Q are:

53, q=21+132+1,22+1-42+153, q=2+33,4-4353, q=53,0353, q=53,0q=0

Hence,  the values of p and is 73 and 0, respectively.

Page No 288:

Question 29:

The line joining the points (2, 1) and (5, −8) is trisected at the points P and Q. If point P lies on the line 2xy + k = 0. Find the value of k.

Answer:

We have two points A (2, 1) and B (5,−8). There are two points P and Q which trisect the line segment joining A and B.

Now according to the section formula if any point P divides a line segment joining andin the ratio m: n internally than,

The point P is the point of trisection of the line segment AB. So, P divides AB in the ratio 1: 2

Now we will use section formula to find the co-ordinates of unknown point A as,

Therefore, co-ordinates of point P is(3,−2)

It is given that point P lies on the line whose equation is

So point A will satisfy this equation.

So,

Page No 288:

Question 30:

Find the ratio in which the line 2x + 3y – 5 = 0 divides the line segment joining the points (8, –9) and (2, 1). Also, find the coordinates of the point of division.

Answer:

Given that, a line 2x + 3y – 5 = 0 divides the line segment joining the points A(8, –9) and B(2, 1). Let the point that divides AB in the ratio k : 1 be X(a, b).

Now,
 Coordinates of X=kx2+x1k+1,ky2+y1k+1=2k+8k+1,k-9k+1

These coordinates satisfy the equation of the line 2x + 3y – 5 = 0.
So,
22k+8k+1+3k-9k+1-5=04k+16+3k-27-5k-5=02k-16=0k=8

Thus, the point X divides the line in the ratio 8 : 1.

Since k=8, the coordinates of point X are:
Coordinates of X=2×8+88+1,8-98+1=16+89,-19=249,-19=83,-19

Hence, the coordinates of the point of division is 83,-19.

Page No 288:

Question 31:

A point P divides the line segment joining the points A(3, −5) and B(−4, 8) such that APPB=k1. If P lies on the line x + y = 0, then find the value of k.                                                                                                                                                                         [CBSE 2012]

Answer:

It is given that APPB=k1.

So, P divides the line segment joining the points A(3, −5) and B(−4, 8) in the ratio k : 1.

Using the section formula, we get

Coordinates of P = -4k+3k+1,8k-5k+1

Since P lies on the line x + y = 0, so

-4k+3k+1+8k-5k+1=0-4k+3+8k-5k+1=04k-2=0k=12

Hence, the value of k is 12.

Page No 288:

Question 32:

The mid-point P of the line segment joining the points A(−10, 4) and B(−2, 0) lies on the line segment joining the points C(−9, −4) and D(−4, y). Find the ratio in which P divides CD. Also, find the value of y.                                                                               [CBSE 2014]

Answer:

It is given that P is the mid-point of the line segment joining the points A(−10, 4) and B(−2, 0).

∴ Coordinates of P = -10+-22,4+02=-122,42=-6,2

Suppose P divides the line segment joining the points C(−9, −4) and D(−4, y) in the ratio k : 1.

Using section formula, we get

Coordinates of P = -4k-9k+1,ky-4k+1

-4k-9k+1,ky-4k+1=-6,2-4k-9k+1=-6 and ky-4k+1=2

Now,

-4k-9k+1=-6-4k-9=-6k-62k=3k=32

So, P divides the line segment CD in the ratio 3 : 2.

Putting k = 32 in ky-4k+1=2, we get
3y2-432+1=23y-85=23y-8=103y=18y=6

Hence, the value of y is 6.

Page No 288:

Question 33:

If the point C -1, 2 divides internally the line segment joining the points  A (2, 5)  and B( xy ) in the ratio 3 : 4 , find the value of x2 + y2 .

Answer:

It is given that the point C(–1, 2) divides the line segment joining the points A(2, 5) and B(xy) in the ratio 3 : 4 internally.
Using the section formula, we get
-1, 2=3×x+4×23+4,3×y+4×53+4-1, 2=3x+87,3y+2073x+87=-1 and 3y+207=2
⇒ 3x + 8 = –7 and 3y + 20 = 14
⇒ 3x = –15 and 3y = –6
x = –5 and y = –2
x2 + y2 = 25 + 4 = 29
Hence, the value of x2 + y2 is 29.

Page No 288:

Question 34:

 ABCD is a parallelogram with vertices A (x1, y1),  B x2, y2, C (x3, y3)  . Find the coordinates  of the fourth vertex D in terms of x1 , x2 , x3, y1, y2 and  y3.

Answer:

Suppose the coordinates of D be (x, y).
Since diagonals of a parallelogram bisect each other.


Therefore the midpoint of AC is the midpoint of BD, i.e x1+x32, y1+y32and x2+x2, y2+y2 respectively.
x1+x3=x2+x and y1+y3=y2+yx=x1+x3-x2  and y=y1+y3-y2Thus coordinates of D are x1+x3-x2, y1+y3-y2 

Page No 288:

Question 35:

The points A x1,y1 , Bx2,y2 , Cx3,y3  are the vertices of  ABC .
(i) The median from A meets BC at D . Find the coordinates of the point  D.
(ii) Find the coordinates of the point P on AD such that AP : PD  = 2 : 1.
(iii) Find the points of coordinates Q and R on medians BE and CF respectively such that BQ : QE = 2 : 1 and CR : RF = 2 : 1.
(iv) What are the coordinates of the centropid of the triangle ABC

Answer:

(i) Median AD of the triangle will divide the side BC in two equal parts.



Therefore, D is the midpoint of side BC.
Coordinates of D are
x2+x32,y2+y32

(ii)
THe point P divided the side AD in the ratio 2: 1.
Coordinates of P are
2×x2+x32+1×x12+1, 2×y2+y32+1×y12+1=x1+x2+x33,y1+y2+y33


(iii)
Median BE of the triangle will divide the side AC in two equal parts.
Therefore, E is the midpoint of side AC.
Coordinates of E are
x1+x32,y1+y32
The point Q divided the side BE in the ratio 2: 1.
Coordinates of Q are
2×x1+x32+1×x22+1, 2×y1+y32+1×y22+1=x1+x2+x33,y1+y2+y33

Similarly, Coordinates of Q are R are x1+x2+x33,y1+y2+y33.

(iv)
The points P, Q and R coincides and is the centroid of the triangle ABC.
So, coordinates of the centroid is x1+x2+x33,y1+y2+y33.

Page No 288:

Question 36:

A (4, 2), B(6, 5) and C (1, 4) are the vertices of ΔABC.

(i) The median from A meets BC in D. Find the coordinates of the point D.

(ii) Find the coordinates of point P on AD such that AP : PD = 2 : 1.

(iii) Find the coordinates of the points Q and R on medians BE and CF respectively such that BQ : QE = 2 : 1 and CR : RF = 2 : 1

(iv) What do you observe?

Answer:

We have triangle in which the co-ordinates of the vertices are A (4, 2); B (6, 5) and C (1, 4)

(i)It is given that median from vertex A meets BC at D. So, D is the mid-point of side BC.

In general to find the mid-point of two pointsand we use section formula as,

Therefore mid-point D of side BC can be written as,

Now equate the individual terms to get,

So co-ordinates of D is

(ii)We have to find the co-ordinates of a point P which divides AD in the ratio 2: 1 internally.

Now according to the section formula if any point P divides a line segment joining andin the ratio m: n internally than,

P divides AD in the ratio 2: 1. So,

(iii)We need to find the mid-point of sides AB and AC. Let the mid-points be F and E for the sides AB and AC respectively.

Therefore mid-point F of side AB can be written as,

So co-ordinates of F is

Similarly mid-point E of side AC can be written as,

So co-ordinates of E is

Q divides BE in the ratio 2: 1. So,

Similarly, R divides CF in the ratio 2: 1. So,

(iv)We observe that that the point P, Q and R coincides with the centroid. This also shows that centroid divides the median in the ratio 2: 1.



Page No 290:

Question 1:

Find the centroid of the triangle whose vertices are:

(i) (1, 4) (−1,−1), (3, −2)

(ii) (−2, 3) (2, −1) (4, 0)

Answer:

We know that the co-ordinates of the centroid of a triangle whose vertices are is-

(i) The co-ordinates of the centroid of a triangle whose vertices are (1, 4); (−1,−1); (3,−2) are-

(ii) The co-ordinates of the centroid of a triangle whose vertices are (−2, 3); (2,−1); (4, 0) are-

Page No 290:

Question 2:

Two vertices of a triangle are (1, 2), (3, 5) and its centroid is at the origin. Find the coordinates of the third vertex.

Answer:

We have to find the co-ordinates of the third vertex of the given triangle. Let the co-ordinates of the third vertex be.

The co-ordinates of other two vertices are (1, 2) and (3, 5)

The co-ordinate of the centroid is (0, 0)

We know that the co-ordinates of the centroid of a triangle whose vertices are is−

So,

Compare individual terms on both the sides-

So,

Similarly,

So,

So the co-ordinate of third vertex



Page No 291:

Question 3:

Find the third vertex of a triangle, if two of its vertices are at (−3, 1) and (0, −2) and the centroid is at the origin.

Answer:

We have to find the co-ordinates of the third vertex of the given triangle. Let the co-ordinates of the third vertex be.

The co-ordinates of other two vertices are (−3, 1) and (0, −2)

The co-ordinate of the centroid is (0, 0)

We know that the co-ordinates of the centroid of a triangle whose vertices are is-

So,

Compare individual terms on both the sides-

So,

Similarly,

So,

So the co-ordinate of third vertex

Page No 291:

Question 4:

A (3, 2) and B (−2, 1)  are two vertices of a triangle ABC whose centroid G has the coordinates 53,-13. Find the coordinates of the third vertex C of the triangle.

Answer:

We have to find the co-ordinates of the third vertex of the given triangle. Let the co-ordinates of the third vertex be.

The co-ordinates of other two vertices are A (3, 2) and C (−2, 1)

The co-ordinate of the centroid is

We know that the co-ordinates of the centroid of a triangle whose vertices are is-

So,

Compare individual terms on both the sides-

So,

Similarly,

So,

So the co-ordinate of third vertex

Page No 291:

Question 5:

If (−2, 3), (4, −3) and (4, 5) are the mid-points of the sides of a triangle, find the coordinates of its centroid.

Answer:

Letbe ant triangle such that P (−2, 3); Q (4,−3) and R (4, 5) are the mid-points of the sides AB, BC, CA respectively.

We have to find the co-ordinates of the centroid of the triangle.

Let the vertices of the triangle be

In general to find the mid-point of two pointsand we use section formula as,

So, co-ordinates of P,

Equate the x component on both the sides to get,

…… (1)

Similarly,

…… (2)

Similarly, co-ordinates of Q,

Equate the x component on both the sides to get,

…… (3)

Similarly,

…… (4)

Similarly, co-ordinates of R,

Equate the x component on both the sides to get,

…… (5)

Similarly,

…… (6)

Add equation (1) (3) and (5) to get,

Similarly, add equation (2) (4) and (6) to get,

We know that the co-ordinates of the centroid G of a triangle whose vertices are is-

So, centroid G of a triangle is,



Page No 302:

Question 1:

Find the area of a triangle whose vertices are

​​(i) (–5, 7), (–4, –5) and (4, 5)

(ii) (1, –1), (–4, 6) and (–3, –5)

Answer:

Area of the triangle formed by the vertices x1, y1, x2, y2 and x3, y3 is 12x1y2-y3+x2y3-y1+x3y1-y2.

(i) Now, the given vertices are (–5, 7), (–4, –5) and (4, 5).

Therefore,
Area of triangle=12-5-5-5+-45-7+47--5                      =12-5-10+-4-2+412                      =1250+8+48    =12106                      =53

Hence, the area of a triangle is 53 square units.

(ii) Now, the given vertices are  (1, –1), (–4, 6) and (–3, –5).


Therefore,
Area of triangle=1216--5+-4-5--1+-3-1-6                      =1216+5+-4-5+1+-3-7                      =1211+-4-4+21                      =1211+16+21                      =1248                      =24

Hence, the area of a triangle is 24 square units.

Page No 302:

Question 2:

​Find the area of the quadrilaterals, the coordinates of whose vertices are

(i) (−3, 2), (5, 4), (7, −6) and (−5, −4)

(ii) (−4, −2), (−3, −5), (3, −2), (2, 3)

Answer:


(i) Let the vertices of the quadrilateral be A (−3, 2), B (5, 4), C (7, −6), and D (−5, −4). Join AC to form two triangles ΔABC and ΔACD.

 

(ii) Let the vertices of the quadrilateral be A (−4, −2), B (−3, −5), C (3, −2), and D (2, 3). Join AC to form two triangles ΔABC and ΔACD.

Page No 302:

Question 3:

The vertices of ΔABC are (−2, 1), (5, 4)  and (2, −3)  respectively. Find the area of the triangle and the length of the altitude through A.

Answer:

GIVEN: The vertices of triangle ABC are A (−2, 1) and B (5, 4) and C (2, −3)

TO FIND: The area of triangle ABC and length if the altitude through A

PROOF: We know area of triangle formed by three points is given by

Now Area of ΔABC

Taking three points A (−2, 1) and B (5, 4) and C(2, −3)

We have

Now,

Page No 302:

Question 4:

If the point P (m, 3) lies on the line segment joining the points A-25,6 and B (2, 8), find the value of m.

Answer:

The formula for the area ‘A’ encompassed by three points, and is given by the formula,

=12x1y2+x2y3+x3y1-x2y1+x3y2+x1y3

If three points are collinear the area encompassed by them is equal to 0.

It is said that the point P(m,3) lies on the line segment joining the points and B(2,8). Hence we understand that these three points are collinear. So the area enclosed by them should be 0.
=12-25×3+m×8+2×6-m×6+2×3+-25×8 0=12-65+8m+12-6m+6-165 0=122m+8 0=2m+8 m=-4

Hence the value of ‘m’ for which the given condition is satisfied is.

Page No 302:

Question 5:

If (x, y) be on the line joining the two points (1, −3) and (−4, 2) , prove that x + y + 2= 0.

Answer:

Since the point (x, y) lie on the line joining the points (1, −3) and (−4, 2); the area of triangle formed by these points is 0.

That is,

Thus, the result is proved.

Page No 302:

Question 6:

Find the value of ​so that the area of triangle ABC with A(k + 1, 1), B(4, –3) and C(7, –k) is 6 square units.

Answer:

Given that, the three vertices of a triangle are A(k + 1, 1), B(4, –3) and C(7, –k).

arABC=12x1y2-y3+x2y3-y1+x3y1-y2
6=12k+1-3+k+4-k-1+71--3=12k+1-3+k+4-k-1+7×4=12-3k-3+k2+k-4k-4+28=12k2-6k+21

12=k2-6k+21k2-6k+9=0k-32=0k=3

Hence, k = 3.

 

Page No 302:

Question 7:

If A(−3, 5), B(−2, −7), C(1, −8) and D(6, 3) are the vertices of a quadrilateral ABCD, find its area.                          [CBSE 2014]

Answer:

It is given that A(−3, 5), B(−2, −7), C(1, −8) and D(6, 3) are the vertices of a quadrilateral ABCD.



Area of the quadrilateral ABCD = Area of ∆ABC + Area of ∆ACD

arABC=12x1y2-y3+x2y3-y1+x3y1-y2                  =12-3-7--8+-2-8-5+15--7                  =12-3+26+12                  =352square units

arACD=12x1y2-y3+x2y3-y1+x3y1-y2                  =12-3-8-3+13-5+65--8                  =1233-2+78                  =1092square units

∴ Area of the quadrilateral ABCD = 352+1092=1442=72 square units

Hence, the area of the given quadrilateral is 72 square units.

Page No 302:

Question 8:

For what value of a the point (a, 1), (1, −1), and (11, 4) are collinear?

Answer:

The formula for the area ‘A’ encompassed by three points, and is given by the formula,
=12x1y2+x2y3+x3y1-x2y1+x3y2+x1y3

If three points are collinear the area encompassed by them is equal to 0.

The three given points are A(a, 1), B(1, −1) and C(11, 4). It is also said that they are collinear and hence the area enclosed by them should be 0.
=12x1y2+x2y3+x3y1-x2y1+x3y2+x1y3  0=12a×-1+1×4+11×1-1×1+11×-1+a×4   0=12-a+4+11-1-11+4a   0=12-a+15--10+4a   0=12-a+15+10-4a   0=12-5a+25   0=-5a+25 5a=25   a=5

Hence the value of ‘a’ for which the given points are collinear is.

Page No 302:

Question 9:

​If the vertices of a triangle are (1, −3), (4, p) and (−9, 7) and its area is 15 sq. units, find the value(s) of p.

Answer:

Let A(1, −3), B(4, p) and C(−9, 7) be the vertices of the ∆ABC.

Here, x1 = 1, y1 = −3; x2 = 4, y2 = and x3 = −9, y3 = 7

ar(∆ABC) = 15 square units

12x1y2-y3+x2y3-y1+x3y1-y2=15121p-7+47--3+-9-3-p=1512p-7+40+27+9p=1510p+60=30

10p+60=30 or 10p+60=-30

10p=-30 or 10p=-90

p=-3 or p=-9

Hence, the value of is −3 or −9.

Page No 302:

Question 10:

​Find the area of triangle whose vertices are:

(i) (at12, 2at1), (at22,2at2) and (at32,2at3)

(ii) (a, c + a), (a, c) and (−a, c − a)

Answer:

Area of triangle formed by three points is given by .

(i) The vertices are given as (at12, 2at1), (at22,2at2) and (at32,2at3).


(ii) The vertices are given as .

Page No 302:

Question 11:

In  ABC , the coordinates of vertex A are (0, -1) and D (1,0) and E(0,10)  respectively the mid-points of the sides AB and AC . If F is the mid-points of the side BC , find the area of  DEF.

Answer:



Let the coordinates of B and C be x2, y2 and x3, y3, respectively.

D is the midpoint of AB.

So,

1, 0=x2+02, y2-121=x22 and 0=y2-12x2=2 and y2=1

Thus, the coordinates of B are (2, 1).

Similarly, E is the midpoint of AC.

So,

0, 1=x3+02, y3-120=x32 and 1=y3-12x3=0 and y3=3

Thus, the coordinates of C are (0, 3).

Also, F is the midpoint of BC. So, its coordinates are 

2+02, 1+32=1, 2

Now,

Area of a triangle = 12x1y2-y3+x2y3-y1+x3y1-y2

Thus, the area of ABC is 

1201-3+23+1+0-1-1=12×8=4 square units

And the area of DEF is

1211-2+02-0+10-1=12×-2=1 square unit     Taking the numerical value, as the area cannot be negative

Page No 302:

Question 12:

Find the area of the triangle PQR with Q(3, 2) and the mid-points of the sides through Q being (2, −1) and (1, 2).            [CBSE 2015]

Answer:

Let P(x1, y1), Q(3, 2) and R(x2, y2) be the vertices of the ∆PQR.

Suppose S(2, −1) and T(1, 2) be the mid-points of sides QR and PQ, respectively.



Using mid-point formula, we have

x1+32=1, y1+22=2x1+3=2, y1+2=4x1=-1, y1=2

∴ Coordinates of P = (−1, 2)

Also,

x2+32=2, y2+22=-1x2+3=4, y2+2=-2x2=1, y2=-4

∴ Coordinates of R = (1, −4)

So, P(−1, 2), Q(3, 2) and R(1, −4) are the vertices of ∆PQR.

arPQR=12x1y2-y3+x2y3-y1+x3y1-y2                      =12-12--4+3-4-2+12-2                      =12-6-18+0                      =12-24                      =12×24                      =12 square units
                   
Hence, the area of the triangle is 12 square units.

Page No 302:

Question 13:

If R (x, y) is a point on the line segment joining the points P (a, b) and Q (b, a), then prove that x + y = a + b.

Answer:

The formula for the area ‘A’ encompassed by three points, and is given by the formula,

=12x1y2+x2y3+x3y1-x2y1+x3y2+x1y3

If three points are collinear the area encompassed by them is equal to 0.

It is said that the point R(x, y) lies on the line segment joining the points P(a, b) and Q(b, a). Hence we understand that these three points are collinear. So the area enclosed by them should be 0.
=12ay+xa+b2-xb+by+a2 0=12ay+xa+b2-xb-by-a2 0=ay+xa+b2-xb-by-a2 a2-b2=ax+ay-bx-by a+ba-b=a-bx+y a+b=x+y

Hence under the given conditions we have proved that.

Page No 302:

Question 14:

Find the value of a for which the area of the triangle formed by the points A(a, 2a), B(−2, 6) and C(3, 1) is 10 square units.

Answer:

The formula for the area ‘A’ encompassed by three points, and is given by the formula,

=12x1y2+x2y3+x3y1-x2y1+x3y2+x1y3

The three given points are A(a,2a), B(2,6) and C(3,1). It is also said that the area enclosed by them is 10 square units. Substituting these values in the above mentioned formula we have,

=12a×6+-2×1+3×2a--2×2a+3×6+a×110=126a-2+6a--4a+18+a10=1215a-2020=15a-204=3a-4

We have. Hence either

Or

Hence the values of ‘a’ which satisfies the given conditions are.

Page No 302:

Question 15:

If , prove that the points (a, a2), (b, b2), (c, c2) can never be collinear.

Answer:

GIVEN: If

TO PROVE: that the points can never be collinear.

PROOF:

We know three points are collinear when

Now taking three points

Also it is given that

Hence area of triangle made by these points is never zero. Hence given points are never collinear.

Page No 302:

Question 16:

The area of a triangle is 5 sq units. Two of its vertices are (2, 1) and (3, –2). If the third vertex is 72, y, find the value of y.

Answer:

Let Ax1, y1=A2, 1, Bx2, y2=B3,-2 and Cx3, y3=C72,y.
Now
AreaABC=12x1y2-y3+x2y3-y1+x3y1-y25=122-2-y+3y-1+721+210=-4-2y+3y-3+21210=y+7210=y+72    or -10=y+72 y=132 or y = -272
Hence, y = 132 or -272.

Page No 302:

Question 17:

The point A divides the join of P (−5, 1)  and Q(3, 5) in the ratio k:1. Find the two values of k for which the area of ΔABC where B is (1, 5) and C(7, −2) is equal to 2 units.

Answer:

GIVEN: point A divides the line segment joining P (−5, 1) and Q (3, −5) in the ratio k: 1

Coordinates of point B (1, 5) and C (7, −2)

TO FIND: The value of k

PROOF: point A divides the line segment joining P (−5, 1) and Q (3, −5) in the ratio k: 1

So the coordinates of A are

We know area of triangle formed by three points is given by =12x1y2+x2y3+x3y1-x2y1+x3y2+x1y3

Now Area of ΔABC= 2 sq units.

Taking three points A , B (1, 5) and C (7, −2)

Hence

Page No 302:

Question 18:

The area of a triangle is 5. Two of its vertices are (2, 1) and (3, −2). The third vertex lies on y = x + 3. Find the third vertex.

Answer:

GIVEN: The area of triangle is 5.Two of its vertices are (2, 1) and (3, −2). The third vertex lies on y = x+3

TO FIND: The third vertex.

PROOF: Let the third vertex be (x, y)

We know area of triangle formed by three points is given by =12x1y2+x2y3+x3y1-x2y1+x3y2+x1y3

Now

Taking three points(x, y), (2, 1) and (3, −2)

Also it is given the third vertex lies on y = x+3

Substituting the value in equation (1) and (2) we get

Hence the coordinates of and



Page No 303:

Question 19:

Find the area of a parallelogram ABCD if three of its vertices are A(2, 4), B(2 + 3, 5) and C(2, 6).                 [CBSE 2013]

Answer:

It is given that A(2, 4), B(2 + 3, 5) and C(2, 6) are the vertices of the parallelogram ABCD.



We know that the diagonal of a parallelogram divides it into two triangles having equal area.

∴ Area of the parallogram ABCD = 2 × Area of the ∆ABC

Now,
arABC=12x1y2-y3+x2y3-y1+x3y1-y2                  =1225-6+2+36-4+24-5                  =12-2+4+23-2                  =12×23                  =3square units

∴ Area of the parallogram ABCD = 2 × Area of the ∆ABC = 2 × 3 = 23 square units

Hence, the area of given parallelogram is 23 square units.

Page No 303:

Question 20:

Find the value(s) of k for which the points (3k − 1, k − 2), (k, k − 7) and (k − 1, −k − 2) are collinear.                        [CBSE 2014]

Answer:

Let A(3k − 1, k − 2), B(k, k − 7) and C(k − 1, −k − 2) be the given points.

The given points are collinear. Then,

arABC=012x1y2-y3+x2y3-y1+x3y1-y2=0x1y2-y3+x2y3-y1+x3y1-y2=0
3k-1k-7--k-2+k-k-2-k-2+k-1k-2-k-7=03k-12k-5+k-2k+5k-1=06k2-17k+5-2k2+5k-5=04k2-12k=0
4kk-3=0k=0 or k-3=0k=0 or k=3

Hence, the value of k is 0 or 3.

Page No 303:

Question 21:

If the points A(−1, −4), B(b, c) and C(5, −1) are collinear and 2b + c = 4, find the values of b and c.                      [CBSE 2014]

Answer:

The given points A(−1, −4), B(b, c) and C(5, −1) are collinear.

arABC=012x1y2-y3+x2y3-y1+x3y1-y2=0x1y2-y3+x2y3-y1+x3y1-y2=0
-1c--1+b-1--4+5-4-c=0-c-1+3b-20-5c=03b-6c=21b-2c=7               .....1

Also, it is given that

2b + c = 4               .....(2)

Solving (1) and (2), we get

27+2c+c=414+4c+c=45c=-10c=-2              

Putting c = −2 in (1), we get

b-2×-2=7b=7-4=3

Hence, the respective values of b and c are 3 and −2.

Page No 303:

Question 22:

If ab0, prove that the points (a, a2), (b, b2) (0, 0) will not be collinear.

Answer:

Let A(a, a2), B(b, b2) and C(0, 0) be the coordinates of the given points.
We know that the area of triangle having vertices x1, y1, x2, y2 and x3, y3 is 12x1y2-y3+x2y3-y1+x3y1-y2 square units.
So,
Area of ∆ABC
=12ab2-0+b0-a2+0a2-b2=12ab2-a2b=12abb-a0             ab0
Since the area of the triangle formed by the points (a, a2), (b, b2) and (0, 0) is not zero, so the given points are not collinear.

Page No 303:

Question 23:

Four points A (6, 3), B (−3, 5), C(4, −2) and D (x, 3x) are given in such a way that DBCABC=12, find x.

Answer:

GIVEN: four points A (6, 3), B (−3, 5) C (4, −2) and D(x, 3x) such that

TO FIND: the value of x

PROOF:

We know area of the triangles formed by three points is given by

Now

Area of triangle DBC taking D(x, 3x), B (−3, 5), C (4, −2)

Area of triangle ABC taking, A (6, 3), B (−3, 5), C (4, −2)

Also it is given that

Substituting the values from (1) and (2) we get

Page No 303:

Question 24:

If three points (x1, y1) (x2, y2), (x3, y3) lie on the same line, prove that

y2-y3x2x3+y3-y1x3x1+y1-y2x1x2=0

Answer:

GIVEN: If three points lie on the same line

TO PROVE:

PROOF:

We know that three points are collinear if

Hence proved.

Page No 303:

Question 25:

If the points A(1, -2) , B (2,3) , C (a,2)  and D (-4,-3)  form a parallelogram , find the value of  and height of the parallelogram taking  AB as base .

Answer:

Since diagonals of a parallelogram bisect eachother.
Coordinates of the midpoint of AC = coordinates of the midpoint of BD.
a+12,2-22=-4+22,-3+32a+12,0=-1,0On comparing,a+12=-1a=-3Area of the ABC isA=1213-2+22+2-3-2-3A=121+8+15A=12 sq. units

Since, ABCD is a parallelogram,
Area of ABCD = 2 × area of triangle ABC  = 2 × 12 = 24 sq. units
Height of the parallelogram is area of the parallelogram divided by its base.
Base AB is
AB=1-22+-2-32=12+52=26Height = 2426=12226

Page No 303:

Question 26:

A6,1  , B(8,2)  and C(9, 4)  are three vertices of a parallelogram ABCD . If E is the mid-point  of DC , find the area of  ADE.

Answer:



Three vertices are given, then D can be calulated and it comes out to be (7, 3).
Since, E is midpoint of BD.
Therefore, coordinates of E are 152,52.
Now, vertices of triangle ABE rae (6, 1), (8, 2) and 152,52.
Area of theABE =12161182115252=12120-15-652-2+1152-8=125-62-12=34aq. units

Page No 303:

Question 27:

If D-15,52, E(7,3) and  F72,72 are the mid-points of sides of  ABC ,  find the area of   ABC .

Answer:


The midpoint of BC is D-12,52,
The midpoint of AB is F72,72,
The midpoint of AC is E7,3,
Consider the line segment BC,
p+r2=-12  ;  q+s2=52p+r=-1   ;  q+s=5     .....(i)Consider the line segment AB,p+x2=72  ;  q+y2=72p+x=7  ;  q+y=7     .....(ii)

Consider the line segment AC,r+x2=7  ;  s+y2=3r+x=14  ;  s+y=6     .....(iii)

Solve (i), (ii) and (iii) to get Ax,y=A11,4, Bp,q=B-4,3, Cr,s=C3,2
Let us assume that BC is base of the triangle,
BC=-4-32+3-22=50Equation of the line BC is x+4-4-3=y-33-2x+7y-17=0The perpendicular distance from a point Px1,y1isP=111+74-1750=2250
The area of the triangle is A=12×50×2250=11 sq. units
 



Page No 306:

Question 1:

Find the distance of a point P(x, y) from the origin.

Answer:

The given point is P(xy). 
The origin is O(0, 0).
PO=x2-x12+y2-y12=x-02+y-02=x2+y2
Thus, the distance of point P from the origin is PO = x2+y2 units.

Page No 306:

Question 2:

Write the coordinates of a point P on x-axis which is equidistant from the point A(–2, 0) and B(6, 0).

Answer:

Let coordinates of a point on x-axis be P(x, 0).

Thus, AP = BP
AP2=BP2x--22+0-022=x-62+0-022x+22+02=x-62+02x+22=x-62x2+4+4x=x2+36-12x4x+12x=36-416x=32x=2Therefore, x=2.

Hence, the coordinates of a point on x-axis are (2, 0).

Page No 306:

Question 3:

Find the value(s) of x, if the distance between the points A(0, 0) and B(x, –4) is 5 units.

Answer:

The distance between x1, y1 and x2, y2 is x2-x12+y2-y12.

Therefore, the distance between the points A(0, 0) and B(x, –4)=x-02+-4-02
5=x2+-42Squaring both sides, we get52=x2+16225=x2+16x2+16-25=0x2-9=0x-3x+3=0x=±3

Hence, the value(s) of x are 3 and ​–3.

Page No 306:

Question 4:

If the distance between the points (4, k) and (1, 0) is 5, then what can be the possible values of k?

Answer:

Consider the points A(4, k) and B(1, 0).
It is given that the distance AB is 5 units.
By distance formula, disance AB is as follows:
AB=4-12+k-025=9+k225=9+k216=k2±4=k
Hence, values of k are ±4.

Page No 306:

Question 5:

What is the distance between the points Asinθ-cosθ,0 and B0,sinθ+cosθ?                    [CBSE 2015]

Answer:

The given points are Asinθ-cosθ,0 and B0,sinθ+cosθ.

Using distance formula, we have

AB=sinθ-cosθ-02+0-sinθ+cosθ2      =sinθ-cosθ2+sinθ+cosθ2      =sin2θ+cos2θ-2sinθcosθ+sin2θ+cos2θ+2sinθcosθ      =2sin2θ+cos2θ      =2 units                                          sin2θ+cos2θ=1

Thus, the distance between the given points is 2 units.

Page No 306:

Question 6:

If A (1, 2) B (4, 3) and C (6, 6) are the three vertices of a parallelogram ABCD, find the coordinates of fourth vertex D.

Answer:

Let ABCD be a parallelogram in which the co-ordinates of the vertices are A (1, 2);

B (4, 3) and C (6, 6). We have to find the co-ordinates of the forth vertex.

Let the forth vertex be

Since ABCD is a parallelogram, the diagonals bisect each other. Therefore the mid-point of the diagonals of the parallelogram will coincide.

Now to find the mid-point of two pointsand we use section formula as,

The mid-point of the diagonals of the parallelogram will coincide.

So,

Therefore,

Now equate the individual terms to get the unknown value. So,

Similarly,

So the forth vertex is

Page No 306:

Question 7:

If P (2, p) is the mid-point of the line segment joining the points A (6, −5) and B (−2, 11). find the value of p.

Answer:

It is given that mid-point of line segment joining A (6,−5) and B (−2, 11) is

In general to find the mid-point of two pointsand we use section formula as,

So,

Now equate the y component to get,



Page No 307:

Question 8:

If P (x, 6) is the mid-point of the line segment joining A (6, 5) and B (4, y), find y.

Answer:

It is given that mid-point of line segment joining A (6, 5) and B (4, y) is

In general to find the mid-point of two pointsand we use section formula as,

So,

Now equate the y component to get,

So,

Page No 307:

Question 9:

If the distance between the points (3, 0) and (0, y) is 5 units and y is positive. then what is the value of y?

Answer:

It is given that distance between P (3, 0) and is 5.

In general, the distance between A and B is given by,

So,

On further simplification,

We will neglect the negative value. So,

Page No 307:

Question 10:

If P (2, 6) is the mid-point of the line segment joining A (6, 5) and B (4, y), find y.

Answer:

It is given that mid-point of line segment joining A (6, 5) and B (4, y) is P (2, 6)

In general to find the mid-point of two pointsand we use section formula as,

So,

Now equate the y component to get,

So,

Page No 307:

Question 11:

What is the distance between the points A (c, 0) and B (0, −c)?

Answer:

We have to find the distance between and .

In general, the distance between A and B is given by,

So,

Page No 307:

Question 12:

Find the value of a so that the point (3, a) lies on the line represented by 2x − 3y + 5 = 0

Answer:

If a point is said lie on a line represented by, then the given equation of the line should hold true when the values of the co-ordinates of the points are substituted in it.

Here it is said that the point (3, a) lies on the line represented by the equation.

Substituting the co-ordinates of the values in the equation of the line we have,

Thus the value of ‘a’ satisfying the given conditions is.

Page No 307:

Question 13:

Find the distance between the points -82, 2 and 25, 2

Answer:

We have to find the distance between and .

In general, the distance between A and B is given by,

So,

Page No 307:

Question 14:

Write the ratio in which the line segment doining the points A (3, −6), and B (5, 3) is divided by X-axis.

Answer:

Let P be the point of intersection of x-axis with the line segment joining A (3,−6) and B (5, 3) which divides the line segment AB in the ratio.

Now according to the section formula if point a point P divides a line segment joining andin the ratio m: n internally than,

Now we will use section formula as,

Now equate the y component on both the sides,

On further simplification,

So x-axis divides AB in the ratio 2:1.

Page No 307:

Question 15:

Find the values of x for which the distance between the point P(2, −3), and Q (x, 5) is 10.

Answer:

It is given that distance between P (2,−3) and is 10.

In general, the distance between A and B is given by,

So,

On further simplification,

Page No 307:

Question 16:

Write the condition of collinearity of points (x1, y1), (x2, y2) and (x3, y3).

Answer:

The condition for co linearity of three points, and is that the area enclosed by them should be equal to 0.

The formula for the area ‘A’ encompassed by three points, and is given by the formula,

Thus for the three points to be collinear we need to have,

The area ‘A’ encompassed by three points, and is also given by the formula,

Thus for the three points to be collinear we can also have,

Page No 307:

Question 17:

Write the formula for the area of the triangle having its vertices at (x1, y1), (x2, y2) and (x3, y3).

Answer:

The formula for the area ‘A’ encompassed by three points, and is given by the formula,

The area ‘A’ encompassed by three points, and is also given by the formula,

 

Page No 307:

Question 18:

Write the coordinates of a point on X-axis which is equidistant from the points (−3, 4) and (2, 5).

Answer:

The distance d between two points and is given by the formula

Here we are to find out a point on the x−axis which is equidistant from both the points

A(-3,4) and B(2,5).

Let this point be denoted as C(x, y).

Since the point lies on the x-axis the value of its ordinate will be 0. Or in other words we have.

Now let us find out the distances from ‘A’ and ‘B’ to ‘C’

We know that both these distances are the same. So equating both these we get,

Squaring on both sides we have,

Hence the point on the x-axis which lies at equal distances from the mentioned points is.

Page No 307:

Question 19:

Write the coordinates of the point dividing line segment joining points (2, 3) and (3, 4) internally in the ratio 1 : 5.

Answer:

Let P be the point which divide the line segment joining A (2, 3) and B (3, 4) in the ratio 1: 5.

Now according to the section formula if point a point P divides a line segment joining andin the ratio m: n internally than,

Now we will use section formula as,

So co-ordinate of P is

Page No 307:

Question 20:

If the distance between points (x, 0) and (0, 3) is 5, what are the values of x?

Answer:

We have to find the unknown x using the distance between A and B which is 5.In general, the distance between A and B is given by,

So,

Squaring both the sides we get,

So,

Page No 307:

Question 21:

What is the distance between the points (5 sin 60°, 0) and (0, 5 sin 30°)?

Answer:

We have to find the distance between A and B.

In general, the distance between A and B is given by,

So,

But according to the trigonometric identity,

And,

Therefore,

Page No 307:

Question 22:

Write the ratio in which the line segment joining points (2, 3) and (3, −2) is divided by X axis.

Answer:

Let P be the point of intersection of x-axis with the line segment joining A (2, 3) and B (3,−2) which divides the line segment AB in the ratio.

Now according to the section formula if point a point P divides a line segment joining andin the ratio m: n internally than,

Now we will use section formula as,

Now equate the y component on both the sides,

On further simplification,

So x-axis divides AB in the ratio

Page No 307:

Question 23:

Write the distance between the points A (10 cos θ, 0) and B (0, 10 sin θ).

Answer:

We have to find the distance between A and B.

In general, the distance between A and B is given by,

So,

But according to the trigonometric identity,

Therefore,

Page No 307:

Question 24:

Write the perimeter of the triangle formed  by the points O (0, 0), A (a, 0) and B (0, b).

Answer:

The distance d between two points and is given by the formula

The perimeter of a triangle is the sum of lengths of its sides.

The three vertices of the given triangle are O(0, 0), A(a, 0) and B(0, b).

Let us now find the lengths of the sides of the triangle.

The perimeter ‘P’ of the triangle is thus,

Thus the perimeter of the triangle with the given vertices is.

Page No 307:

Question 25:

If A (−1, 3) , B(1, −1) and C (5, 1) are the vertices of a triangle ABC, what is the length of the median through vertex A?

Answer:

We have a triangle in which the co-ordinates of the vertices are A (−1, 3) B (1,−1) and

C (5, 1). In general to find the mid-point of two pointsand we use section formula as,

Therefore mid-point D of side BC can be written as,

Now equate the individual terms to get,

So co-ordinates of D is (3, 0)

So the length of median from A to the side BC,

Page No 307:

Question 26:

What is the area of the triangle formed by the points O (0, 0), A (6, 0) and B (0, 4)?

Answer:

The given triangle is a right angled triangle, right angled at O. the co-ordinates of the vertices are O (0, 0) A (6, 0) and B (0, 4).

So,

Altitude is 6 units and base is 4 units.

Therefore,

Page No 307:

Question 27:

If the mid-point of the segment joining A (x, y + 1) and B (x + 1, y + 2) is C 32,52, find x, y.

Answer:

It is given that mid-point of line segment joining A and B is C

In general to find the mid-point of two pointsand we use section formula as,

So,

Now equate the components separately to get,

So,

Similarly,

So,

Page No 307:

Question 28:

Two vertices of a triangle have coordinates (−8, 7) and (9, 4) . If the centroid of the triangle is at the origin, what are the coordinates of the third vertex?

Answer:

We have to find the co-ordinates of the third vertex of the given triangle. Let the co-ordinates of the third vertex be.

The co-ordinates of other two vertices are (−8, 7) and (9, 4)

The co-ordinate of the centroid is (0, 0)

We know that the co-ordinates of the centroid of a triangle whose vertices are is-

So,

Compare individual terms on both the sides-

So,

Similarly,

So,

So the co-ordinate of third vertex

Page No 307:

Question 29:

Write the coordinates the reflections of points (3, 5) in X and Y -axes.

Answer:

We have to find the reflection of (3, 5) along x-axis and y-axis.

Reflection of any pointalong x-axis is

So reflection of (3, 5) along x-axis is

Similarly, reflection of any pointalong y-axis is

So, reflection of (3, 5) along y-axis is

Page No 307:

Question 30:

Find the area of triangle with vertices ( a, b+c) , (b, c+a) and (c, a+b).

Answer:

The area ‘A’ encompassed by three points, and is given by the formula,

Here, three points, and are a, b+c, b, c+a and c, a+b.
Area is as follows:
a, b+c, b, c+a and c, a+bA=12ac+a-a-b+ba+b-b-c+cb+c-c-a=12ac-b+ba-c+cb-a=12ac-ab+ba-bc+cb-ca=0

Page No 307:

Question 31:

If the points A (1,2) , O (0,0) and(a,b) are collinear , then find  a : b.

Answer:

For the three points, and to be collinear we need to have area enclosed between the points equal to zero.
Here, points, and are A1,2,O0,0 and Ca,b.
1210-b+0b-2+a2-0=0-b+2a=02a=bab=12

Page No 307:

Question 32:

Find the coordinates of the point which is equidistant from the three vertices A(2x, 0) O (0,0) and B(0,2y) of  AOB .

Answer:



It is known that, in a right angled triangle midpoint of the hypotenuse is equidistant from ots vertices.

Suppose D be the midpoint of the hypotenuse AB.
The coordinates of D are2x+02,0+2y2=x,y.

Page No 307:

Question 33:

If the centroid of the triangle formed by points P (a, b), Q(b, c) and R (c, a) is at the origin, what is the value of a + b + c?

Answer:

The co-ordinates of the vertices are (a, b); (b, c) and (c, a)

The co-ordinate of the centroid is (0, 0)

We know that the co-ordinates of the centroid of a triangle whose vertices are is-

So,

Compare individual terms on both the sides-

Therefore,

Page No 307:

Question 34:

​In Q. No. 33, what is the value of a2bc+b2ca+c2ab?

Answer:

The co-ordinates of the vertices are (a, b), (b, c) and (c, a).

The co-ordinate of the centroid is (0, 0).

We know that the co-ordinates of the centroid of a triangle whose vertices are  is .

So,

Compare individual terms on both the sides, we get

Therefore,

To find the value of

Multiply and divide it by  to get,

Now as we know that if,

Then,

So,

Page No 307:

Question 35:

If points Q and reflections of point P (−3, 4) in X and Y axes respectively, what is QR?

Answer:

We have to find the reflection of (−3, 4) along x-axis and y-axis.

Reflection of any pointalong x-axis is

So reflection of (−3, 4) along x-axis is

Similarly, reflection of any pointalong y-axis is

So, reflection of (−3, 4) along y-axis is

Therefore,

Page No 307:

Question 36:

What are the coordinates of the point where the perpendicular bisector of the line segment joining the points A(1, 5) and B(4, 6) cuts the y-axis?

Answer:

Suppose CD be the perpendicular bisector of the line AB.
Suppose CD intersect AB in E.
Coodinates of E are
4+12,6+52=52,112Let the slope of CD be m, since AB CD.Slope of CD × slope of AB = -1m×6-54-1=-1m=-3equation of CD is y-112=-3x-522y-11=-6x+156x+2y-26=0Equation of perpendicular bisector of line AB is 6x+2y-26=0.Put x = 0 in the above equation.0+2y-26=0y=13

The coordinates are (0, 13).



Page No 308:

Question 1:

The distance of the point (2, 3) from x-axis is  ________.

Answer:

The distance between the point (2, 3) and x-axis can be determined by assuming a point (2, 0) on  x-axis.

The distance between x1, y1 and x2, y2 is x2-x12+y2-y12.

Therefore, the distance between (2, 3) and (2, 0)=2-22+0-32
                                                                              = 32
​                                                                              = 9
​                                                                              = 3

Hence, the distance of the point (2, 3) from x-axis is  3 units.

Page No 308:

Question 2:

The distance of the point (–4, 7) from y-axis is _________.

Answer:

The distance between the point (–4, 7) and y-axis can be determined by assuming a point (0, 7) on y-axis.

The distance between x1, y1 and x2, y2 is x2-x12+y2-y12.

Therefore, the distance between (–4, 7) and (0, 7)=0--42+7-72
                                                                                = -42
​                                                                                = 16
​                                                                                = 4

Hence, the distance of the point (–4, 7) from y-axis is 4 units.

Page No 308:

Question 3:

If the centroid of the triangle whose vertices are (2, 4), (3, a), (4, 2) is (a, 3), then a = _________.

Answer:

Centroid of the triangle with vertices x1, y1, x2, y2 and x3, y3 is x1+x2+x33, y1+y2+y33.Centroid of the triangle with vertices 2, 4, 3, a and 4, 2=a, 32+3+43, 4+a+23=a, 393, 6+a3=a, 33, 6+a3=a, 33=a

Hence, a = 3.

Page No 308:

Question 4:

The distance between the points (1, 0) and (2, cot θ) is _________.

Answer:

The distance between x1, y1 and x2, y2 is x2-x12+y2-y12.

Therefore, the distance between the points (1, 0) and (2, cotθ)=2-12+cotθ-02
                                                                                                   =12+cotθ2=1+cotθ2=Cosecθ2=Cosecθ

Hence, the distance between the points (1, 0) and (2, cotθ) is cosecθ.

Page No 308:

Question 5:

The distance between the points (0, 5) and (5, 0) is __________.

Answer:

The distance between x1, y1 and x2, y2 is x2-x12+y2-y12.

Therefore, the distance between the points (0, 5) and (5, 0)=5-02+0-52
                                                                                              =52+-52=25+25=50=52

Hence, the distance between the points (0, 5) and (5, 0) is 52 units.

Page No 308:

Question 6:

If the distance between the points (2, –2) and (–1, x) is 5, then the values of x are __________.

Answer:

The distance between x1, y1 and x2, y2 is x2-x12+y2-y12.

Therefore, the distance between the points (2, –2) and (–1, x)=-1-22+x--22
5=-32+x+22Squaring both sides, we get52=9+x+22225=9+x+2225=9+x2+4+4xx2+4x+13-25=0x2+4x-12=0x2+6x-2x-12=0xx+6-2x+6=0x-2x+6=0x=2, -6

Hence, the values of x are 2 and ​–6.

Page No 308:

Question 7:

The area of the triangle with vertices at A(3, 0), B(7, 0) and C(8, 4) is ___________.

Answer:

Area of the triangle formed by the vertices x1, y1, x2, y2 and x3, y3 is
12x1y2-y3+x2y3-y1+x3y1-y2

Now, the given vertices are A(3, 0), B(7, 0) and C(8, 4)

Therefore,
Area of triangle=1230-4+74-0+80-0                      =123-4+74+80                      =12-12+28+0                      =1216                      =8

Hence, the area of a triangle is 8 square units.

Page No 308:

Question 8:

The quadrant in which the point dividing the line segment joining the points (7, –6) and (3, 4) internally in the ratio 1 : 2 is _________.

Answer:

Let P(xy) be the point dividing the line segment joining the points (7, –6) and (3, 4) internally in the ratio 1 : 2 

Section formula: if the point (xy) divides the line segment joining the points (x1y1) and (x2y2) internally in the ratio : n, then the coordinates (xy) = mx2+nx1m+n, my2+ny1m+n

Therefore, using section formula, the coordinates of P are:

x, y=13+271+2,14+2-61+2        =3+143,4-123        =173,-83        =173,-83

Hence, the quadrant in which the point dividing the line segment joining the points (7, –6) and (3, 4) internally in the ratio 1 : 2 is IV quadrant.

Page No 308:

Question 9:

If the distance between the points (4, p) and (1, 0) is 5, then the values of p are __________.

Answer:

The distance between x1, y1 and x2, y2 is x2-x12+y2-y12.

Therefore, the distance between the points (4, p) and (1, 0)=1-42+0-p2
5=-32+-p2Squaring both sides, we get52=9+p2225=9+p2p2=25-9p2=16p=±4

Hence, the values of p are 4 and −4.

Page No 308:

Question 10:

If P(a/3,4) is the mid point of the line segment joining the points Q(–6, 5) and R(–2, 3), then the value of 'a' is _________.

Answer:

Mid point of x1, y1 and x2, y2 is x1+x22, y1+y22.

P(a/3,4) is the mid point of the line segment joining the points Q(–6, 5) and R(–2, 3).

Therefore, using mid point formula the coordinates of P are:
a3, 4=-6-22, 5+32a3, 4=-82, 82a3, 4=-4, 4a3=-4a=-12

Hence, the value of 'a' is –12.

Page No 308:

Question 11:

The values of y for which the point (2, –4) is equidistant from the points (3, 8) and (–10, y), are _________.

Answer:

Let the point P(2, –4) is equidistant from the points A(3, 8) and B(–10, y).

Thus, AP = BP
AP2=BP22-32+-4-822=2--102+-4-y22-12+-122=2+102+-4-y21+144=122+4+y21+144=144+16+y2+8y1=16+y2+8yy2+8y+15=0y2+5y+3y+15=0yy+5+3y+5=0y+3y+5=0y=-5, -3Therefore, y=-5, -3.

Hence, the values of y for which the point (2, –4) is equidistant from the points (3, 8) and (–10, y), are –5 and ​–3.

Page No 308:

Question 12:

The x-axis divides the line segment joining the points (–4, –6) and (–1, 7)  in the ratio _________.

Answer:

Section formula: if the point (xy) divides the line segment joining the points (x1y1) and (x2y2) internally in the ratio : 1, then the coordinates (xy) = kx2+x1k+1, ky2+y1k+1

Let the point P(x, 0) divides the line segment joining the points (–4, –6) and (–1, 7) in the ratio : 1.

Therefore, using section formula, the coordinates of P are:

x, 0=k-1+1-4k+1,k7+1-6k+1x, 0=-k-4k+1,7k-6k+1x=-k-4k+1 and 0=7k-6k+1 0=7k-6k+17k-6=07k=6k=67

Hence, the x-axis divides the line segment joining the points (–4, –6) and (–1, 7)  in the ratio 6 : 7.

Page No 308:

Question 13:

If the points (5, 1), (–2, –3) and (8, 2m) are collinear, then m = ___________.

Answer:

If the area of the triangle formed by three points is equal to zero, then the points are collinear.

Area of the triangle formed by the vertices x1, y1, x2, y2 and x3, y3 is 12x1y2-y3+x2y3-y1+x3y1-y2.

Now, the given points (5, 1), (–2, –3) and (8, 2m) are collinear.

Therefore, Area of triangle formed by them is equal to zero.

Area of triangle=0125-3-2m+-22m-1+81--3=012-15-10m-4m+2+81+3=0-13-14m+84=0-13-14m+32=0-14m+19=019-14m=014m=19m=1914

Hence, m = 1914.

Page No 308:

Question 14:

The image of the point (3, –5) in the x-axis has the coordinates __________.

Answer:

The image of (x, y) in the x-axis is (x, –y).

Thus, the image of the point (3, –5) in the x-axis is (3, 5).

Hence, the image of the point (3, –5) in the x-axis has the coordinates (3, 5).

Page No 308:

Question 15:

The coordinates of the image of the point (–4, 5) in y-axis are _________.

Answer:

The image of (xy) in the y-axis is (–xy).

Thus, the image of the point (–4, 5) in y-axis is (4, 5).

Hence, the coordinates of the image of the point (–4, 5) in y-axis are (4, 5).

Page No 308:

Question 16:

The x-coordinate of the point lying on the perpendicular bisector of the line segment joining the points A(–2, –5) and B(2, 5) is_______.

Answer:

Perpendicular bisectors divides the line segment in two equal parts.

Mid point of x1, y1 and x2, y2 is x1+x22, y1+y22.


Therefore, the mid point of A(–2, –5) and B(2, 5)
=-2+22, -5+52=02, 02=0, 0

Hence, the x-coordinate of the point lying on the perpendicular bisector of the line segment joining the points A(–2, –5) and B(2, 5) is 0.

Page No 308:

Question 17:

If the distance between the points A(–3, –14) and B(a, –5) is 9 units, then a = __________.

Answer:

The distance between x1, y1 and x2, y2 is x2-x12+y2-y12.

Therefore, the distance between the points A(–3, –14) and B(a, –5)=a--32+-5--142
9=a+32+-5+142Squaring both sides, we get92=a+32+92281=a+32+81a+32=0a=-3

Hence, –3.

Page No 308:

Question 18:

The ratio in which the point P34, 512 divides the line segment joining the points A(1/2, 3/2) and B(2, –5) is _________.

Answer:

Section formula: if the point (xy) divides the line segment joining the points (x1y1) and (x2y2) internally in the ratio : 1, then the coordinates (xy) = kx2+x1k+1, ky2+y1k+1

Let the point P34, 512 divides the line segment joining the points A(1/2, 3/2) and B(2, –5) in the ratio : 1.

Therefore, using section formula, the coordinates of P are:

34, 512=k2+112k+1,k-5+132k+134, 512=2k+12k+1,-5k+32k+134, 512=4k+12k+1,-10k+32k+134, 512=4k+12k+2,-10k+32k+24k+12k+2=3444k+1=32k+216k+4=6k+610k=2k=210k=15

Hence, the ratio in which the point P34, 512 divides the line segment joining the points A(1/2, 3/2) and B(2, –5) is 1 : 5.

Page No 308:

Question 19:

The number of points on x-axis which are at a distance of 25 from the point (7, –4), is __________.

Answer:

The distance between x1, y1 and x2, y2 is x2-x12+y2-y12.

Let the point on x-axis be P(x, 0) which is at a distance of 25 from the point (7, –4).

Therefore, the distance between the points (x, 0) and (7, –4)=7-x2+-4-02
25=7-x2+-42Squaring both sides, we get252=7-x2+16220=7-x2+1620=49+x2-14x+16x2-14x+65-20=0x2-14x+45=0x2-9x-5x+45=0xx-9-5x-9=0x-5x-9=0x=5, 9

Hence, the number of points on x-axis which are at a distance of 25 from the point (7, –4), is 2.

Page No 308:

Question 20:

It (2, –2) and (5, 2) are opposite vertices of a square, then the length of the side of the square is ___________.

Answer:

Length of the opposite vertices of a square = Length of the diagonal

 Length of the diagonal=5-22+2--22                                    =32+2+22                                    =32+42                                    =9+16                                    =25                                    =5

But, Length of the diagonal = 2 (Length of the side of the square)

 5=2×Length of the sideLength of the side=52                              =52×22                              =522

Hence, the length of the side of the square is 522 units.



Page No 309:

Question 21:

The coordinates of the point which is equidistant from the vertices of the triangle formed by the points O(0, 0), A(a, 0) and B(0, b), are _________.

Answer:

Circumcentre is the point which is equidistant from the vertices of the triangle.

Let the point be C(x, y).

Thus, OC = AC = BC
OC2=AC2=BC2Now, OC2=AC2x-02+y-022=x-a2+y-022x-02+y-02=x-a2+y-02x2+y2=x-a2+y2x2=x2+a2-2ax2ax=a22x=ax=a2Also, OC2=BC2x-02+y-022=x-02+y-b22x-02+y-02=x-02+y-b2x2+y2=x2+y-b2y2=y2+b2-2by2by=b22y=by=b2Therefore, x=a2 and y=b2.

Hence, the coordinates of the point which is equidistant from the vertices of the triangle formed by the points O(0, 0), A(a, 0) and B(0, b), are a2, b2.
 

Page No 309:

Question 22:

If the points (a, 0), (0, b) and (1, 1) are collinear, then 1a+1b = ___________.

Answer:

If the area of the triangle formed by three points is equal to zero, then the points are collinear.

Area of the triangle formed by the vertices x1, y1, x2, y2 and x3, y3 is 12x1y2-y3+x2y3-y1+x3y1-y2.

Now, the given points (a, 0), (0, b) and (1, 1) are collinear.

Therefore, Area of triangle formed by them is equal to zero.

Area of triangle = 012ab-1+01-0+10-b=012ab-a+0-b=0ab-a-b=0ab-a-b=0ab=a+ba+b=abDividing both sides by ab, we getaab+bab=abab1b+1a=1

Hence, 1a+1b =  1.

Page No 309:

Question 23:

If the distance of the point (4, a) from x-axis is half its distance from y-axis, then ___________.

Answer:

The distance between x1, y1 and x2, y2 is x2-x12+y2-y12.

The distance between the point (4, a) and x-axis can be determined by assuming a point (4, 0) on x-axis.

Therefore, the distance between (4, a) and (4, 0)=4-42+0-a2
                                                                             =a2=a

Also, the distance between the point (4, a) and y-axis can be determined by assuming a point (0, a) on y-axis.

Therefore, the distance between (4, a) and (0, a)=4-02+a-a2
                                                                             =42=4

According to the question,
Distance of the point (4, a) from x-axis is half its distance from y-axis.
a=124a=2

Hence, if the distance of the point (4, a) from x-axis is half its distance from y-axis, then the value of a is 2.

Page No 309:

Question 24:

The coordinates of the point equidistant from the vertices O(0, 0), A(6, 0) and B(0, 8) of ΔOAB are ___________.

Answer:

Circumcentre is the point which is equidistant from the vertices of the triangle.

Let the point be C(xy).

Thus, OC = AC = BC
OC2=AC2=BC2Now, OC2=AC2x-02+y-022=x-62+y-022x-02+y-02=x-62+y-02x2+y2=x-62+y2x2=x2+36-12x12x=36x=3Also, OC2=BC2x-02+y-022=x-02+y-822x-02+y-02=x-02+y-82x2+y2=x2+y-82y2=y2+64-16y16y=64y=4Therefore, x=3 and y=4.

Hence, coordinates of the point equidistant from the vertices O(0, 0), A(6, 0) and B(0, 8) of ΔOAB are (3, 4).

Page No 309:

Question 25:

If the centroid of the triangle formed by the points (a, b), (1, a) and (b, 1) is at the origin, then a3+b3+1ab= ____________.

Answer:

Centroid of the triangle with vertices x1, y1, x2, y2 and x3, y3 is x1+x2+x33, y1+y2+y33.Centroid of the triangle with vertices a, b, 1, a and b, 1=0, 0a+1+b3, b+a+13=0, 0a+b+13=0a+b+1=0a+b=-1Now,a3+b3+1ab=a+b3-3aba+b+1ab                 =-13-3ab-1+1ab       a+b=-1                 =-1+3ab+1ab                 =3abab                 =3

Hence,  a3+b3+1ab=3.



View NCERT Solutions for all chapters of Class 10